You are on page 1of 32

Volume 32 Number 5 May 2018

Bitter Pill
Geriatric patients can be particularly complex to manage,
as they often present with multiple comorbidities for which
they are taking a number of medications. More than 90%
of elderly patients take at least one prescription drug (the
average is four), and 75% take at least one over-the-counter
medication. Indiscriminate prescribing practices often
lead to treatment overlap in this vulnerable population,
a complication that can increase the risk of drug-drug
interactions and adverse drug events.

Out on a Limb
The differential diagnosis for a limping child is quite
broad, ranging from simple musculoskeletal etiologies
such as sprains and contusions to serious infections
and malignancies. There are several important
factors that must be considered when evaluating such
cases, including the age of the child and particular
developmental milestones. An appropriate application
of historical, physical, laboratory, and imaging findings
can help determine which children can be safely
discharged with appropriate follow-up care, and which
require hospital admission.

THE OFFICIAL CME PUBLICATION OF THE AMERICAN COLLEGE OF EMERGENCY PHYSICIANS


IN THIS ISSUE
Lesson 9 n The Limping Child . . . . . . . . . . . . . . . . . . . . . . . . . . . . . . . . . . 3
LLSA Literature Review . . . . . . . . . . . . . . . . . . . . . . . . . . . . . . . . . . . . . . . . . 15 Critical Decisions in Emergency Medicine is the official
CME publication of the American College of Emergency
Critical Image . . . . . . . . . . . . . . . . . . . . . . . . . . . . . . . . . . . . . . . . . . . . . . . . . . 16 Physicians. Additional volumes are available.
Critical Procedure . . . . . . . . . . . . . . . . . . . . . . . . . . . . . . . . . . . . . . . . . . . . . . 18
EDITOR-IN-CHIEF
Lesson 10 n Adverse Drug Events in Geriatric Patients . . . . . . . . . . . . 19 Michael S. Beeson, MD, MBA, FACEP
Northeastern Ohio Universities,
Critical ECG . . . . . . . . . . . . . . . . . . . . . . . . . . . . . . . . . . . . . . . . . . . . . . . . . . . 29 Rootstown, OH
CME Questions . . . . . . . . . . . . . . . . . . . . . . . . . . . . . . . . . . . . . . . . . . . . . . . . 30
SECTION EDITORS
Drug Box/Tox Box . . . . . . . . . . . . . . . . . . . . . . . . . . . . . . . . . . . . . . . . . . . . . 32 Andrew J. Eyre, MD
Brigham & Women’s Hospital/Harvard Medical School,
Boston, MA
Contributor Disclosures. In accordance with the ACCME Standards for Commercial
Support and policy of the American College of Emergency Physicians, all individuals with Joshua S. Broder, MD, FACEP
control over CME content (including but not limited to staff, planners, reviewers, and Duke University, Durham, NC
authors) must disclose whether or not they have any relevant financial relationship(s) to Frank LoVecchio, DO, MPH, FACEP
learners prior to the start of the activity. These individuals have indicated that they have Maricopa Medical Center/Banner Phoenix Poison
a relationship which, in the context of their involvement in the CME activity, could be and Drug Information Center, Phoenix, AZ
perceived by some as a real or apparent conflict of interest (eg, ownership of stock, grants,
Amal Mattu, MD, FACEP
honoraria, or consulting fees), but these individuals do not consider that it will influence
University of Maryland, Baltimore, MD
the CME activity. Joshua S. Broder, MD, FACEP; GlaxoSmithKline; his wife is employed by
GlaxoSmithKline as a research organic chemist. All remaining individuals with control over Lynn P. Roppolo, MD, FACEP
CME content have no significant financial interests or relationships to disclose. University of Texas Southwestern Medical Center,
Dallas, TX
This educational activity consists of two lessons, a post-test, and evaluation questions;
as designed, the activity should take approximately 5 hours to complete. The participant Christian A. Tomaszewski, MD, MS, MBA, FACEP
should, in order, review the learning objectives, read the lessons as published in the print University of California Health Sciences,
or online version, and complete the online post-test (a minimum score of 75% is required) San Diego, CA
and evaluation questions. Release date May 1, 2018. Expiration April 30, 2021. Steven J. Warrington, MD, MEd
Accreditation Statement. The American College of Emergency Physicians is accredited Orange Park Medical Center, Orange Park, FL
by the Accreditation Council for Continuing Medical Education to provide continuing
medical education for physicians. ASSOCIATE EDITORS
Wan-Tsu W. Chang, MD
The American College of Emergency Physicians designates this enduring material for a
University of Maryland, Baltimore, MD
maximum of 5 AMA PRA Category 1 Credits™. Physicians should claim only the credit
commensurate with the extent of their participation in the activity. Walter L. Green, MD, FACEP
University of Texas Southwestern Medical Center,
Each issue of Critical Decisions in Emergency Medicine is approved by ACEP for 5 ACEP Dallas, TX
Category I credits. Approved by the AOA for 5 Category 2-B credits.
John C. Greenwood, MD
Commercial Support. There was no commercial support for this CME activity. University of Pennsylvania, Philadelphia, PA
Target Audience. This educational activity has been developed for emergency physicians. Danya Khoujah, MD
University of Maryland, Baltimore, MD
Critical Decisions in Emergency Medicine is a trademark owned and published monthly by the American Sharon E. Mace, MD, FACEP
College of Emergency Physicians, PO Box 619911, Dallas, TX 75261-9911. Send address changes and Cleveland Clinic Lerner College of Medicine
comments to Critical Decisions in Emergency Medicine, PO Box 619911, Dallas, TX 75261-9911, or to
/Case Western Reserve University, Cleveland, OH
cdem@acep.org; call toll-free 800-798-1822, or 972-550-0911.
Jennifer L. Martindale, MD, MSc
Copyright 2018 © by the American College of Emergency Physicians. All rights reserved. No part of this
publication may be reproduced, stored, or transmitted in any form or by any means, electronic or mechanical,
Mount Sinai St. Luke’s/Mount Sinai West,
including storage and retrieval systems, without permission in writing from the Publisher. Printed in the USA. New York, NY

The American College of Emergency Physicians (ACEP) makes every effort to ensure that contributors to its
George Sternbach, MD, FACEP
publications are knowledgeable subject matter experts. Readers are nevertheless advised that the statements Stanford University Medical Center, Stanford, CA
and opinions expressed in this publication are provided as the contributors’ recommendations at the time
of publication and should not be construed as official College policy. ACEP recognizes the complexity of RESIDENT EDITOR
emergency medicine and makes no representation that this publication serves as an authoritative resource
Nathaniel Mann, MD
for the prevention, diagnosis, treatment, or intervention for any medical condition, nor should it be the basis
for the definition of or standard of care that should be practiced by all health care providers at any particular
Massachusetts General Hospital, Boston, MA
time or place. Drugs are generally referred to by generic names. In some instances, brand names are added
for easier recognition. Device manufacturer information is provided according to style conventions of the EDITORIAL STAFF
American Medical Association. ACEP received no commercial support for this publication. Rachel Donihoo, Managing Editor
To the fullest extent permitted by law, and without rdonihoo@acep.org
limitation, ACEP expressly disclaims all liability for Suzannah Alexander, Publishing Assistant
errors or omissions contained within this publication,
and for damages of any kind or nature, arising out of Lexi Schwartz, Subscriptions Coordinator
use, reference to, reliance on, or performance of such Marta Foster, Director, Educational Products
information.
ISSN2325-0186(Print) ISSN2325-8365(Online)
Out on a Limb
The Limping Child

LESSON 9

By Mike A. Gardiner, MD, FACEP, FAAP; Coburn H. Allen, MD,


FAAP, FACEP, FPIDS; Robert Vezzetti, MD, FAAP, FACEP;
and Gael Lonergan, MD
Dr. Gardiner is an assistant clinical professor at the University of California, San
Diego School of Medicine and Rady Children’s Hospital – San Diego. Dr. Allen is the
director of the Pediatric Emergency Medicine Fellowship Program and an associate
professor of pediatrics; Dr. Vezzetti is an assistant professor of pediatrics; and Dr.
Lonergan is an attending physician in the Departments of Pediatric Radiology and
Pediatric Emergency Medicine at the University of Texas Dell Medical School –
Austin, Dell Children’s Medical Center of Central Texas.

Reviewed by Sharon E. Mace, MD, FACEP

OBJECTIVES
On completion of this lesson, you should be able to: CRITICAL DECISIONS
1. Identify common causes of limping in pediatric n What common etiologies should be considered when
patients. evaluating an afebrile child with a limp, and how
2. Evaluate appropriate imaging modalities and should they be managed?
laboratory tests for a limping child. n What etiologies should be suspected in a febrile
3. Describe the optimal treatment for a limping child. child with a limp, and how should they be
4. Understand which pediatric patients can be safely addressed?
discharged from the emergency department and n How and when should imaging be used to assess a
which require admission. limping child?
n What laboratory studies are most valuable in the
FROM THE EM MODEL evaluation of a limping child?
1.0 Signs, Symptoms, and Presentations n When does a pediatric limp warrant inpatient
1.3 General admission?

The differential diagnosis for a limping child is quite broad, ranging from simple musculoskeletal etiologies
such as sprains and contusions to serious infections and malignancies. There are several important factors that
must be considered when evaluating this common presentation, including the age of the patient and particular
developmental milestones.1 An appropriate application of historical, physical, laboratory, and imaging findings
can help determine which children can be safely discharged with appropriate follow-up care, and which require
hospital admission.

May 2018 n Volume 32 Number 5 3


CASE PRESENTATIONS
■ CASE ONE refused to do either for the past 3 days. fever. She is an active soccer player
She denies trauma, cough, congestion, and explains that her trainer has been
A 15-month-old boy presents with
vomiting, or any other symptoms, but “working on” her hip during physical
a limp. His mother states that he was
“jumping around” at a playground mentions decreased oral intake. The therapy sessions, but has now advised
earlier in the day, but began favoring patient was previously healthy and is her to see a physician. The patient
his right leg soon after they arrived fully immunized. Her vital signs are was seen several times by her primary
home. No trauma is noted, and the blood pressure 100/65, pulse rate 160, care provider, who obtained several
patient has not had a fever or recent respiratory rate 30, and temperature
plain radiographs of the knee, all of
illness. He is otherwise healthy and 39°C (102.2°F).
which were normal. Although she has
fully immunized. The patient is irritable and clings
continued to remain active, the pain
His vital signs are blood pressure to her mother. Her ears, nose, and
has begun to interfere with her daily
90/60, pulse rate 110, respiratory throat are normal. Her chest is clear,
and aside from mild tachycardia, activities and she is starting to feel
rate 25, and temperature 36.8°C
her heart appears normal and is “depressed.”
(98.2°F). He is boisterous and
well perfused. The child’s abdomen Her vital signs are blood pressure
active, and his physical examination
reveals a full range of motion of the is soft and nontender. She is grossly 110/70, pulse rate 65, respiratory rate
lower extremities at the hip, knee, neurologically intact but prefers not 16, and temperature 36.8°C (98.2°F).
and ankle joints, without evident to be moved; while she is lying on the She does not appear to be in distress,
erythema, edema, crepitus, or examination bed, she screams and holds until she is asked to ambulate. She
deformity. There is no ecchymosis, both lower extremities flexed at the walks with a limp, complaining of
and there are no lesions. He is hip. She screams even louder when her left-knee pain. She is somewhat obese
grossly neurovascularly intact. When legs are manipulated, but it is difficult and tells you that she is in the 90th
prompted to walk to his mother, the to tell which leg is causing her pain. percentile for weight, but is “not fat.”
child does so readily, but with a slight She holds both legs slightly abducted Her pelvis is stable to compression.
limp to his right. When his right tibia and externally rotated, but the position
There are no indications of trauma;
and fibula are palpated, he begins to appears more pronounced on the right.
no lesions; and no erythema, edema,
cry and withdraws the leg.
■ CASE THREE crepitus, or warmth in the hip,
■ CASE TWO A 16-year-old girl presents with left- knees, or lower extremities. She has a
A 12-month-old girl is brought knee pain. She says her symptoms began complete range of motion in the right
in for fever, fussiness, and a refusal about 6 months ago, but can’t recall any extremity, but complains of pain with
to walk. The mother explains that preceding trauma. She is in generally manipulation of the left femur at the
the child is normally active, cruising good health, and denies any recent hip. Her knee shows no abnormalities,
and beginning to take steps, but has illness or other symptoms, including and she is neurovascularly intact.

CRITICAL DECISION evaluating any limping child, with or can lead to plastic bony deformities
without a history of fever. and unique injuries, including Torus
What common etiologies should
fractures (buckling deformities due
be considered when evaluating Traumatic Etiologies
to compressive load) and Greenstick
an afebrile child with a limp, and A history of trauma primarily raises
fractures (the fracture of one cortex with
how should they be managed? concern for soft-tissue injury and
plastic deformity of the other). Any child
fractures. Several important skeletal
The potential causes of limping in with a traumatic mechanism of injury
differences between adults and children
an afebrile child are quite broad, with and a limp should undergo a thorough
make pediatric trauma unique. The bones
diagnoses ranging from benign to life- physical examination of both legs and
threatening (Table 1). The most common of growing children have an open physis
the spine to determine locations of
culprits include traumatic, skeletal, (“growth plate”), which is the weakest
tenderness, swelling, deformity, warmth,
oncological, infectious, and inflammatory part of the bone (Figure 1). At certain
or erythema.
etiologies. It is important to note, stages of bony development, the physeal
however, that children with transient portions are weaker than the ligaments Toddler’s Fracture
synovitis, osteomyelitis, septic arthritis, supporting them, making fractures more Toddler’s fractures are spiral fractures
and other infectious or inflammatory likely than sprains or strains. of the tibia that occur in ambulatory
conditions may present without fever. It Pediatric bones are also more porous children between the ages of 9 and 36
is crucial to consider these causes when and elastic than adult bones, which months, often following minimal or

4 Critical Decisions in Emergency Medicine


unwitnessed trauma. Children with this through manipulation of the foot with Nonaccidental Trauma
type of injury often present with an acute the knee held stable.2 Plain-film x-rays When a fracture is encountered in
limp and may not bear weight on the often show a nondisplaced or minimally any pediatric patient, particularly infants
affected limb. The physical examination displaced spiral fracture through the mid and developmentally delayed children,
may be normal, or the patient may exhibit to distal tibial shaft. These fractures can it is crucial for emergency providers to
subtle signs of tenderness and/or minimal be quite subtle on frontal and/or lateral ask themselves whether the fracture may
swelling in the mid to distal tibia. views, and are best visualized using the be the result of nonaccidental trauma.
Pain may be elicited through direct internal oblique view. Child abuse is an under-recognized
palpation over the fracture site, by passive As many as 40% of children with cause of pediatric trauma, and missed or
dorsiflexion of the ipsilateral foot, or toddler’s fractures have normal x-rays delayed identification may lead to further
by applying a twisting force to the tibia on initial evaluation; however, a fracture morbidity and mortality.5 Any fracture
may be presumed if the history and in a nonambulatory child should raise
TABLE 1. Differential Diagnoses physical examination are consistent concern for inflicted injury. It is crucial
of Pediatric Limping with this diagnosis and the child is to obtain thorough details about the
afebrile. Due to the minimal trauma purported mechanism of injury.
Traumatic Etiologies
required to cause a toddler’s fracture, It is helpful to understand several
Fracture
this diagnosis can also be made based on general gross motor milestones in normal
Sprain or strain
characteristic radiographic findings, even infant development when evaluating for
Nonaccidental trauma
Overuse injury
without a history of witnessed trauma. possible inflicted injury. For instance,
Apophysitis The treatment for a toddler’s fracture infants typically begin rolling over at 3 to
Soft-tissue injury or foreign body involves immobilization with a posterior 4 months of age; therefore, a 2-month-
long-leg splint and orthopedic follow-up old infant presenting with a fracture
Skeletal Etiologies
for casting. Repeated imaging after 7 to after “rolling off the bed” should arouse
Slipped capital femoral epiphysis
10 days may show periosteal formation suspicion. When encountering injuries
Legg-Calvé-Perthes disease
Osteochondroma in cases of occult fracture with initially in children who are developmentally
Osteochondritis dessicans normal x-rays.2,3 delayed, the proposed mechanism must be
Developmental dysplasia of the hip Compartment Syndrome taken into account in the context of the
Tarsal coalition Any limping child with a traumatic patient’s developmental state.
Leg-length discrepancy Red flags that may indicate
mechanism of injury should be evaluated
Blounts disease
for possible neurovascular compromise nonaccidental trauma include a significant
Oncological Etiologies and compartment syndrome. The clinician delay in seeking care, other evidence
Osteoid osteoma should assess the severity of pain, of injury such as bruising or patterned
Bone cyst sensation, strength, and distal pulses.1 injuries (eg, belt marks or cigarette burns),
Ewing sarcoma The disease can be caused by traumatic and multiple fractures in varied stages
Osteosarcoma of healing. Several fracture types that
injuries, but also may occur as a result
Metastatic bone tumor
of vascular insult, infection, malignancy, should raise suspicion for inflicted injury
Leukemia
or other atraumatic etiologies.4 The include metaphyseal fractures such as
Infections most common fractures leading to corner fractures, bucket-handle fractures
Septic arthritis compartment syndrome in children are (Figure 2), sternal fractures, scapular
Osteomyelitis tibial shaft, humeral shaft, and forearm fractures, spinous process fractures, rib
Discitis fractures (particularly posterior ribs),
fractures, with tibial shaft fractures
Myositis
accounting for more than 40% of cases. femur fractures in nonambulatory
Spinal epidural abscess
The classic “five Ps” of compartment children, bilateral long bone fractures,
Intra-abdominal infection
syndrome (pain, paresthesia, paralysis, vertebral body fractures, digital fractures,
Noninfectious Inflammatory Conditions and complex skull fractures.6 While none
pallor, and pulselessness) are often seen
Transient synovitis late, and may be absent or not assessable of these abnormalities are pathognomonic
Juvenile idiopathic arthritis for nonaccidental trauma, metaphyseal,
in pediatric patients. Other signs and
Henoch-Schönlein purpura
symptoms that should increase suspicion sternal, scapular, posterior rib, and
Spondyloarthropathy
for this diagnosis include pain out of spinous process fractures are highly
Systemic lupus erythematosus
Acute appendicitis proportion to the injury, pain with passive specific for abuse.
stretch, or the “three As” (increasing The American Association of
Other Etiologies
anxiety, agitation, and analgesic Pediatrics recommends a skeletal survey
Muscular dystrophy
requirement). If compartment syndrome is for the evaluation of suspected abuse in
Neuropathy
suspected, an orthopedic surgeon should children younger than 2 years; the test is
Spinal cord lesion or compression
be consulted emergently; measurement indicated on a case-by-case basis in those
Reflex sympathetic dystrophy
Conversion disorder of compartment pressures and/or a between the ages of 2 and 5 years.7 The
fasciotomy may be considered. initial survey may be repeated 10 days

May 2018 n Volume 32 Number 5 5


later to evaluate for occult fracture. Note Overuse Injuries Sindig-Larsen-Johansson disease (inferior
that skeletal surveys add little additional Overuse and apophyseal injuries patellar apophysitis). These diagnoses
diagnostic value in patients older than 5 frequently happen in adolescents aged 8 manifest with pain and tenderness
years. to 15 years following periods of new or isolated to the tendon insertion site at the
Other studies that are used to evaluate intense activity. Although stress fractures heel, tibial tuberosity, and inferior pole of
cases of suspected abuse vary depending commonly occur in the distal tibia or the patella, respectively.
on each clinical scenario and local foot, they can be found anywhere along X-rays are often normal or show
practice policy. Potential studies include the lower extremity; x-ray findings may only soft-tissue swelling; however, subtle
a noncontrast head CT, evaluation of be subtle or delayed. Focal tenderness bony changes occasionally appear at the
transaminase levels, urinalysis with with or without a palpable callus site of tendon insertion. Overuse and
microscopic evaluation, coagulation formation can indicate an overuse injury, apophyseal injuries are initially managed
studies, toxicology studies, a calcium and particularly in patients with a compatible conservatively with rest, nonsteroidal
vitamin D panel, and an ophthalmological history. anti-inflammatory medications
evaluation for retinal hemorrhages. Apophyseal injuries are caused by (NSAIDs), and stretching of the affected
A consultation with child abuse inflammation at the point where major muscles. Cases that do not respond
pediatricians (where available) and/or the tendons are inserted into bone. Common to conservative therapy often can be
involvement of Child Protective Service causes include Sever disease (calcaneal managed with periods of immobilization
agencies can help guide appropriate apophysitis), Osgood-Schlatter disease through splinting and/or casting.
management and follow-up care. (tibial tuberosity apophysitis), and Orthopedic surgery or sports medicine
clinic follow-up also is important.

FIGURE 1. Normal Knee of a Pediatric Patient Skeletal Etiologies


Slipped Capital Femoral Epiphysis
Children and adolescents, particularly
those who are overweight or obese, may
suffer from a condition known as slipped
capital femoral epiphysis (SCFE). This is
a disorder in which the capital femoral
epiphysis becomes displaced from the
femoral neck through the physis. While
most children with SCFE present with
hip pain or a chronic limp, thigh or knee
pain is the presenting symptom in as
many as 15% of cases.8 Most patients
complain of a dull aching pain in the hip,
groin, thigh, or knee that is worsened by
activity and relieved with rest.
Although the symptoms generally
are chronic, some patients present
with acute complaints, or following
trauma. SCFE occurs most commonly
in patients between the ages of 10 and
16 years, with obesity, male gender,
and underlying endocrinopathies (eg,
hypothyroidism, growth hormone
restriction, or panhypopituitarism) as
significant risk factors.9,10 On physical
examination, patients with a unilateral
SCFE often demonstrate out-toeing of
the affected leg, an antalgic gait, variable
hip tenderness, and decreased internal
rotation, abduction, and flexion of the
hip with pain during these maneuvers.
MRI of a child’s normal knee (lateral view), showing the open growth plates at the end of Patients with suspected SCFE should
the femur (upper bone) and the upper part of the tibia (lower bone). The growth plates
(yellow bands) cross horizontally and add length to the long bones. The brown-orange
undergo radiographs of the hips and
ends of the bones are epiphyses, and the blue portion is the diaphysis. pelvis. Because a single anteroposterior
(AP) view can miss subtle cases, a lateral

6 Critical Decisions in Emergency Medicine


FIGURE 2. Classic Metaphyseal Bucket-Handle Fracture FIGURE 3. Legg-Calvé-Perthes
Disease

The left distal femur shows an acute transverse metaphyseal fracture in a 4-week-old infant.
AP view of the left hip in a 4-year-old boy
Metaphyseal fractures are also seen in the left distal tibia. Note the periosteal reaction
with Legg-Calvé-Perthes disease. The left
along the shafts of the left tibia and fibula.
femoral head is irregular, flattened, and
IMAGE COURTESY OF DR. THOMAS SANCHEZ/EMRES.COM sclerotic.

view of the hips should also be obtained, controversial, and practice patterns LCP typically present with an insidious
including a frog-leg view. Typical are variable.10,11 Patients with acute onset of chronic hip pain or a limp that
radiographic findings include superior (symptoms <3 weeks) or unstable is worsened by activity. In many cases,
and anterolateral displacement of the (non-weightbearing or with complete limping is the more prominent symptom,
femoral metaphysis relative to the capital dissociation between the epiphysis and with minimal or no complaints of
epiphysis. In a normal hip radiograph, metaphysis) require emergent pinning. discomfort.
a line drawn along the superior femoral Such patients also should be designated Occasionally, pediatric patients
neck (Klein’s line) intersects the lateral as strictly non-weightbearing and experience stress fractures of the hip
portion of the femoral head. In cases denoted as “nothing by mouth” or nil per with routine or minimal trauma, which
of SCFE, Klein’s line does not intersect os (NPO) in preparation for immediate may lead to the diagnosis of LCP. The
the epiphysis, or is asymmetrical when surgical intervention. physical examination may reveal an
compared with the unaffected hip. Patients with chronic but stable antalgic gait to the affected limb with
Other radiographic findings may SCFE should also be designated as non- limited passive abduction and internal
include widening, increased lucency, weightbearing to prevent further slip; rotation of the hip, with or without pain
and irregularity of the affected physis. surgical fixation may be delayed at the or tenderness. Plain films early in the
These additional findings may be the discretion of the orthopedic surgeon. disease course are often normal, with
only identifiable abnormalities in cases Children with SCFE need long-term disease progression showing a small,
of “preslip,” in which the displacement is orthopedic follow-up, as they are at high dense, fragmented capital epiphysis and a
minimal or not yet apparent. If suspicion risk for osteonecrosis of the affected hip widened, irregular physis (Figure 3). MRI
for SCFE persists despite normal or and development of contralateral SCFE. may show bone marrow changes and can
inconclusive x-rays, magnetic resonance Legg-Calvé-Perthes aid in diagnosis if plain films are normal.
imaging (MRI) can be used to detect Another common skeletal cause of an Patients with LCP should avoid
radiographically occult fractures. afebrile limp is idiopathic osteonecrosis bearing weight and should be referred to
The treatment of SCFE requires of the hip, also known as Legg-Calvé- an orthopedic surgeon. Such cases can
surgical pinning of the epiphysis in the Perthes disease (LCP). LCP has a typical often be managed nonsurgically with
current location at the time of diagnosis onset of symptoms between 3 and 12 splinting to maintain the femoral head
without reduction, as attempts at years of age (with the peak between 5 within the acetabulum. Younger patients
reduction have been correlated with and 7 years of age). The disease is four with mild disease do very well; when
worse outcomes. The decision to pin times more common in males than in treated conservatively, children younger
the contralateral hip prophylactically females, and 10% to 20% of cases are than 6 years often make a full recovery
in patients with unilateral SCFE is bilateral at presentation.12 Children with without long-term deficits.13 Patients

May 2018 n Volume 32 Number 5 7


who fail conservative therapies may The staging and treatment of malignant time (eg, staphylococcal osteomyelitis
require surgical osteotomy to obtain primary bone tumors is beyond the with adjacent myositis and/or septic
proper healing and alignment of the scope of this article; however, further arthritis). Pyogenic processes tend to
femoral head. discussion with a pediatric oncologist is be confined and therefore create focal/
recommended. unilateral signs and symptoms, whereas
Oncological Etiologies
Acute Leukemia viral and noninfectious causes of fever
A variety of tumors — both benign
A recent meta-analysis of presenting and limping often present with more
and malignant — commonly present in
signs and symptoms in children with diffuse, symmetric/bilateral signs and
children as bone pain or a limp.
leukemia showed that 43% experience symptoms or difficult-to-localize pain.
Osteoid Osteoma Skin and soft-tissue infections,
bone pain.16 Although patients with
Osteoid osteoma is most common in including cellulitis, boils, adenitis, and
leukemia commonly present with several
teenagers (70% of cases are diagnosed in paronychia, are readily identified with a
complaints, pain and/or a limp can be
those younger than 20 years). Symptoms thorough examination of an undressed
the only presenting symptoms. Related
include focal nocturnal pain and aching patient. Special attention should be paid
findings include pallor, petechiae,
that responds well to NSAIDs. Although to the feet, looking closely at and between
ecchymosis, and hepatosplenomegaly; a
these benign lesions often affect long the toes and the plantar surface for signs
thorough physical examination is crucial
bones, they can erupt anywhere in of puncture wounds and foreign bodies.
in cases of atraumatic bony pain or
the skeleton.14 While the physical Boils have a propensity to develop on the
limping.17
examination is often normal, it may buttocks and perineal skin of children,
Radiographic findings in leukemia
reveal focal swelling, muscle atrophy or and can easily be missed through
are variable, but may include osteopenia,
contractures, or leg-length discrepancy. misguided attempts to respect the privacy
metaphyseal bands, lytic lesions, sclerotic
Osteoid osteomas are often apparent of patients.
on plain-film radiographs as a bony changes, and bony destruction. Although
Gram-positive bacteria (eg, Strepto­
lucency with a dense central nidus; the management of acute leukemia
coccus pyogenes and Staphylo­coccus
however, MRI occasionally is required is beyond the scope of this article,
aureus) are almost always the cause of
to make a definitive diagnosis. Treatment consultation with a pediatric oncologist is
skin and soft-tissue infections; however,
consists of supportive care and pain crucial for further evaluation.
Gram-negative bacteria and anaerobes
management with NSAIDs, as lesions should be considered in puncture
CRITICAL DECISION
often resolve spontaneously over wounds, infected foreign bodies, and in
months to years. Patients with severe What etiologies should be
any case that includes perianal/perineal
pain or tumors that do not improve suspected in a febrile child with involvement.
spontaneously may require surgical a limp, and how should they be Children suffer from higher rates
excision or radiofrequency ablation of addressed? of bone and joint infections, most
the nidus. These procedures may be commonly of the leg, than adult
curative, but recurrence is a common Fever and extremity injury/pain are
the two most frequent, chief complaints patients.19 The majority of osteoarticular
complication.14 infections are acquired through the
of children seen in the emergency
Malignant Tumors bloodstream (hematogenous) but may be
department, according to a survey of
The two most common malignant more than 600 facilities in the United the result of trauma or chronic decubitus
bone tumors in children are Ewing States.18 Infection is usually to blame in wounds (especially pelvic and ankle/
sarcoma and osteosarcoma. These febrile children who present with a limp; foot) with direct inoculation of the
tumors often occur in the distal femur bone or joint. Osteomyelitis is clinically
however, other serious inflammatory
and proximal tibia, but may manifest characterized by fever and focal bone
etiologies include rheumatological and
anywhere along the axial or appendicular tenderness, usually over the metaphysis
oncological diseases. A thorough history,
skeleton.15 They may be diagnosed in long bones.
physical examination, and testing, when
incidentally on x-rays following minor Children with bony involvement
appropriate, allow the clinician to quickly
trauma, or may trigger an acute visit from the foot to the spine may report
ascertain the most likely culprits.
for chronic pain, limping, or other pain, a limp, and an inability to bear
musculoskeletal complaints. Infections weight; infants may simply be fussy.
Pain that occurs at night, at rest, or A broad number of pathogens Similar features occur in joint infections;
in nonarticular areas, as well as systemic (typically viral or bacterial) have the however, the tenderness is focused
symptoms, should raise suspicion for propensity to cause extremity pain and around a joint in such cases, and mobility
malignancy. These tumors are often fever. Extremity pain and limping may is restricted and extremely painful.
diagnosed on x-rays and show poorly originate from infections that involve Although bone and joint infections may
defined tumor margins and characteristic different types of tissues, including skin, manifest overriding warmth, redness, and
findings, including “onion skinning” in bone, joint, muscle, and vertebral disc swelling, these symptoms tend to be less
cases of Ewing sarcoma and a “sunburst” tissues. Infections frequently occur in definitive than in children with cellulitis,
pattern in osteosarcoma (Figure 4). more than one tissue type at the same adenitis, or boils. Likewise, pediatric bone

8 Critical Decisions in Emergency Medicine


and joint infections rarely develop from (hemoglobinopathies) are recovered at toe walking. Bacterial myositis is usually
an overriding skin or soft-tissue infection. increased rates in select populations. focal; it commonly involves the upper leg
Vertebral, pelvic, and hip infections Polymicrobial infections are expected or deep pelvic muscles, or erupts adjacent
are subtler in presentation than those in patients with chronic wounds and to an underlying osteomyelitis.
involving the knee, ankle, and foot. in cases of trauma. Lyme disease also Careful examination of the involved
Osteomyelitis with concomitant adjacent should be considered in any child with area may reveal asymmetric swelling,
septic arthritis occurs mostly in young arthritis and fever who has been exposed warmth, and more diffuse tenderness
infants, in whom the venous connections to a Lyme-endemic area.20 than osteoarticular infections. There
between the metaphysis and epiphysis Myositis (“pyomyositis” if focal is typically minimal to no overriding
allow the spread of bacteria toward the purulent collection) can be bacterial or redness, and features are even more subtle
joint. These “bridging vessels” cease to viral in origin. Viral myositis is fairly when deep pelvic muscles are involved.
exist by 18 to 24 months of age. common in children with influenza (but As with other bacterial causes of pediatric
The predominant organism implicated many other viruses have been implicated fever and limping, Gram-positive bacteria
in osteoarticular infections for all ages to a lesser degree) and has an interesting predominate; antistaphylococcal coverage
and bone types is Staphylococcus aureus. predisposition for the calf muscles, is warranted.21
However, Gram-negative bacteria such leading to a classic “Charlie Chaplin” Vertebral disc infections, or “discitis,”
as Kingella kingae (in preschoolers gait (toes pointed out, shuffling to avoid can lead to fever (often remote) and
with septic arthritis) and Salmonella stretching the gastrocnemius muscles) or limping (or not bearing weight) in young

FIGURE 4. Osteosarcoma FIGURE 5. Osteomyelitis

X-ray of the left knee reveals osteosarcoma of the upper part


of the tibia, with a rupture of the cortical bone. X-ray reveals acute osteomyelitis in an 11-year-old child.

May 2018 n Volume 32 Number 5 9


children, and are particularly easy however, in a child with hip pain, fever, CRITICAL DECISION
to miss. Discitis has a more indolent and/or refusal to bear weight, these tests
How and when should imaging
presentation than osteoarticular and may be necessary to differentiate TS
muscle infections. Special attention to
be used to assess a limping child?
from septic arthritis. In such cases, plain
the lower spine, especially tenderness films generally are normal, ultrasound It is important to limit radiation
with percussion of the lumbar area, may may show a hip effusion, and MRI may exposure as much as possible when
be the only localizing finding. Infants demonstrate a hip effusion with synovial assessing a pediatric patient, regardless
may also prefer to lie down instead of enhancement and the absence of marrow of which imaging modality is selected.
to sit, and may roll over and move more signal enhancement. This goal can be achieved without
gingerly than usual. The etiology of Pediatric rheumatological and onco-­­ compromising study integrity or
discitis is seldom discovered; however, ­­logical conditions that may present as usefulness. Improved clinical decision
Staphylococcus aureus is the organism fever and limping include juvenile rules, the use of ultrasound and MRI,
most often recovered.22 idiopathic arthritis (JIA), dermatomy­ lower-dose CT protocols, and a general
awareness of the effects of ionizing
Noninfectious Inflammatory ositis, systemic lupus erythematosus
radiation have led to a promulgation
Conditions (SLE), rheumatic fever, and leukemia.
of the concept “as low as reasonably
In a large surveillance study of children
Transient synovitis (toxic synovitis) achievable.”24,25 The American College
is a more common cause of limping with acute musculoskeletal complaints,
of Radiology appropriateness guidelines,
in children between the ages of 3 and only 0.1% of cases were attributed to
which are an excellent resource for
8 years than infection. This condition noninfectious inflammatory arthritis,
managing a limping child, compare
often causes acute hip pain, limping, or whereas nearly 90% were due to
imaging modalities to specific clinical
refusal to bear weight, with or without trauma.23
scenarios to help the clinician make
a fever.1 Transient synovitis (TS) is a Generally, children with these
informed decisions.
postinfectious autoimmune inflammatory conditions have important historical
condition that commonly occurs after and examination findings that suggest Plain Radiography
a viral or group A strep infection; a chronic systemic illness, and most Plain radiographs are an appropriate
however, in many cases, no specific cause complain of bilateral symmetrical or first-line imaging choice for assessing
is identified. TS is a self-limited disease multifocal pain that may be migratory a limping child, especially in cases of
that often resolves without incident. or wax and wane. Additionally, these trauma. X-rays provide low doses of
Supportive care with NSAIDs is generally cases typically are accompanied by more radiation, are readily available, and do not
sufficient for pain control. ominous features such as weight loss, typically require sedation. If a specific area
Laboratory work and imaging are organ dysfunction, pallor, and abnormal of concern is identified during the physical
not necessary to diagnose the disease; blood cell lines or counts. examination, initial selected imaging can
be confined to that area.
Unfortunately, this may not always
FIGURE 6. Slipped Epiphysis be possible, particularly in infants and
younger children (ie, <5 years of age). In
such cases, it is not uncommon to obtain
full-extremity x-rays, which may be
defined as AP and lateral views of the tibia
and fibula, and dedicated AP and lateral
views of the femur. This may also include
views of the pelvis or foot.27 The increased
number of views is understandable given
that most young children are unable to
accurately articulate the origin of their
pain. However, studies indicate that
the tibia is, by far, the most common
bone injured in cases of lower-extremity
trauma, especially in young children
who are non-weightbearing and without
focality on their physical examination (ie,
no apparent edema or localizing pain).
This suggests that initial imaging can
be limited to the tibia of the involved side
in patients who present in this manner;
X-ray reveals slipped capital femoral epiphysis in an 11-year-old girl.
additional imaging can be dictated by

10 Critical Decisions in Emergency Medicine


with hip pain. Classic examples include available, rapid, painless, and free of
FIGURE 7. Toddler’s Fracture a slipped capital femoral epiphysis ionizing radiation, the modality has
(Figure 6) or Legg-Calvé-Perthes disease, been advocated in both atraumatic and
both of which can be detected by plain traumatic clinical scenarios. Ultrasound
radiography and are clinical entities that is excellent for the evaluation of
should not be missed. A frog-leg view joint disease (eg, developmental or
should be obtained in such cases, as rheumatological conditions), and is also
SCFE — in particular — can be subtle extremely sensitive for the detection of
and may not be apparent on a single AP hip effusion and fractures.30
view. Because SCFE and LCP can also Ultrasound has several limitations that
be bilateral, a complete pelvic view is should be considered, however, including
required (not just the portion of the hip its inability to distinguish the underlying
in question).29 Neoplasms, congenital cause of a hip effusion. If performed too
malformations, and other findings such as early or inadequately, the test can lead to
bone cysts can also be revealed by plain false negatives.31 This presents a clinical
radiography. conundrum: If the study is positive, is
It is important to employ proper the effusion the result of an infection or
technique when imaging any patient. The another etiology? And can a negative
evaluation should include two complete study be falsely reassuring, leading to a
views (AP and lateral) of the extremity missed pathology?
in question. In most cases, comparison
Magnetic Resonance Imaging
views of the contralateral extremity
MRI lacks ionizing radiation and
are unnecessary; however, they may
provides superior resolution for the
be helpful if questions exist about the
Radiograph of the right tibia and fibula in evaluation of a number of clinical
normalcy of a particular finding/variant.
a 16-month-old girl shows a nondisplaced conditions. However, there are several
oblique fracture through the mid- Remember that not all fractures can be
disadvantages that should be considered,
diaphysis of the right tibia (yellow arrow). seen on two-view imaging; a third view
including cost. Given the time and
may be required. A common example of
technique required to complete a
this is the toddler’s fracture (Figure 7),
proper MRI, some children may require
historical or clinical findings.27 It is which may not be apparent on AP or
sedation. Since contrast material is often
important to note, however, that if initial lateral views but is readily seen on an
used, intravenous access may also be
imaging fails to reveal a pathology, oblique view. A good rule of thumb is to
necessary.
follow-up imaging is recommended. start with two views of the extremity, and
Although the modality is sensitive
Periosteal reactions are part of the normal then add an oblique view if warranted
for traumatic soft-tissue and bone
fracture evolution process and can be by clinical suspicion for an extremity
injuries, it is not typically employed as
reliable indicators of injury. fracture.
an initial test.32 If a trauma diagnosis
Plain radiography can reveal When interpreting pediatric x-rays,
is in question or there is concern for
subperiosteal reactions (in the case of it is important to keep in mind that
alternative conditions, MRI may be
osteomyelitis) or joint-space widening mineralization and ossification vary
appropriate. In children with hip pain,
(in the case of a septic joint), findings across the age spectrum. A consultation
suspected inflammatory conditions (eg,
that may prompt more definitive imaging with a radiologist can help the clinician
osteomyelitis or myositis), or concern for
studies. X-rays are recommended for determine if a questionable finding
malignancy, MRI is a superior imaging
the initial evaluation of suspected is pathological or a normal variant.
choice.
osteomyelitis (Figure 5) or joint Additionally, it is extremely beneficial to
MRI is very good at detecting joint or
provide the reading radio­logist with as
infections, principally to rule out other cartilage inflammation as well as erosion
many historical and physical examination
etiologies such as unreported trauma, or loss of joint space, so it can be useful
details as possible. It is important
malignancies, or previously undetected for the evaluation of malignancies and
to remember that pediatric injuries,
congenital abnormalities. However, an rheumatological conditions.33 Moreover,
particularly epiphyseal and buckle
appreciable length of time (≥2 weeks) it is the preferred definitive imaging
fractures, can be extremely subtle.
is required for such changes to appear; modality in any suspected case of
radiographs alone should not be used to Ultrasonography osteomyelitis, abscess, or pyomyositis.
eliminate this process, especially if the Ultrasonography has increased in Interestingly, the use of contrast
index of suspicion for disease is high.28 popularity in recent years as an imaging when evaluating a possible osseous
Plain radiographs can also be useful modality with multiple applications in infection does not appear to increase
when assessing a limp of nontraumatic the evaluation of a number of pediatric sensitivity or specificity; however, it does
origin. This is especially true of patients conditions, including limping. Readily appear to heighten the confidence of the

May 2018 n Volume 32 Number 5 11


interpreting radiologist, and has the are not a first-line imaging choice in from other types of crisis (high count).
added benefit of identifying the extent the evaluation of a limping child and PCR testing is helpful in determining
of infection or abscess.34 Consequently, are rarely employed in the emergency infectious causes of culture-negative
MRI (with and without intravenous department. osteoarticular infections; specimens
contrast) is usually performed when containing purulent material obtained in
evaluating for infection or inflammatory CRITICAL DECISION the emergency department can be held
processes. What laboratory studies are most in the laboratory for molecular testing if
valuable in the evaluation of a cultures remain negative after several days.
Computed Tomography
Children with suspected rheumatic
In most instances, CT does not play limping child?
fever should undergo throat swabbing
an immediate role in the evaluation of Most cases of pediatric limping do for rapid testing and culture, as well as
a limping child. This modality has the not warrant a laboratory evaluation; blood testing for streptococcal antibodies
advantages of being widely available however, children with fever (eg, ASO). Antinuclear antibodies and
and fairly rapid; however, ionizing (current or remote), prolonged or rheumatoid factor tests are neither
radiation remains a concern. Sedation indolent presentations, comorbidities sensitive nor specific, and are not
may be required to obtain optimal (hemoglobinopathies), or ominous recommended for the routine screening of
images — a process that may necessitate features (ie, weight loss, pallor, and/ children with joint pain. Lyme serology
additional medications (possibly through or toxic appearance) are more likely to testing should only be performed in
intravenous access) and observation have a process that could be identified patients who present with an appropriate
in the emergency department. CT may with simple laboratory tests. In children exposure history.20
be used, however, in cases where a with suspected focal bacterial infection Studies indicate that four criteria (fever
localization of bone abnormalities is with fever, a complete blood count >38.5°C [101.3°F], refusal to bear weight,
indicated (eg, when lesions or masses with differential, a blood culture, an a white blood cell count >12,000/mm3,
are present), or if an intracranial erythrocyte sedimentation rate (ESR) and an ESR >40 mm/hr) can be used
pathology is the suspected cause of a test, and a C-reactive protein (CRP) test to help differentiate transient synovitis
gait disturbance.35 are recommended. from septic arthritis.38,39 In a validation
When plain radiographs are unclear Many experts also recommend study, subjects had a prevalence of septic
and clinical suspicion persists, CT can ordering a complete metabolic profile arthritis ranging from 2% with zero
delineate suspected fractures quite well.36 (ie, a Chem-20) for patients with positive criteria to 93% with four positive
In such cases, a noncontrast study is the suspected infections, comorbidities, or criteria. Further research indicates that
initial technique of choice. A contrast-
chronic or severe presentations; the test a CRP greater than 2.0 mg/dL may be a
enhanced CT scan can be performed if
can identify other organ involvement moderate predictor of septic arthritis with
immediate imaging is warranted and
and help grade disease severity. Children a positive likelihood ratio of 2.9.40 Given
MRI is unavailable or contraindicated
with hemoglobinopathies (eg, sickle-cell the significant morbidity associated with
(eg, in cases of suspected infection or
diseases) should undergo a reticulocyte a delayed diagnosis, children with an
neoplasm).
count to differentiate aplastic (low count) elevated CRP or any of the above criteria
Bone Scan
Bone scans, a radionucleotide
imaging modality, have shown efficacy
in the evaluation of children with
nonspecific limps.37 The whole body can
be imaged using this technique, which is
particularly sensitive to bone and soft- n It is important to consider both benign and life-threatening etiologies when
tissue abnormalities.26 These benefits evaluating a pediatric patient with limping or extremity pain. The child’s
come into play particularly when the history and physical examination should guide the appropriate workup.
physical examination is limited (eg, n Radiography is a reasonable first imaging technique for most cases of
when a child is nonverbal or the history pediatric limping. In younger children with a nonfocal examination, it may
is unclear). be necessary to image the entire extremity.
Disadvantages of the technique n Toddler’s fractures may occur with minimal or unwitnessed trauma to the
include radiation exposure. In addition, lower extremity, and x-ray findings may be subtle or absent on the initial
the test often necessitates admission visit. This diagnosis may be made clinically with normal plain films, or based
or transfer to another institution due solely on x-ray findings without a known mechanism.
to its limited availability. Bone scans n While transient synovitis is a common cause of fever and limping, septic
also take time to complete; typically, arthritis and osteomyelitis may be subtle; providers should maintain a
the test is not initiated for several high index of suspicion for these diagnoses. Laboratory studies are often
hours after the injection of contrast necessary to evaluate a febrile limping child.
material. Radionucleotide bone scans

12 Critical Decisions in Emergency Medicine


should be considered for orthopedic permitted angulation and displacement of worsening pain, particularly if the child
surgery consultation and/or MRI. fractures varies by fracture type, location, is unable to bear weight on the affected
In addition, Kingella kingae leads to and institutional practice. Therefore, in limb at home. Patients with suspicious
less significant elevation of all the noted borderline or atypical fractures, local or borderline laboratory findings, those
laboratory studies and is an increasingly practice guidelines should be reviewed, who do not bear weight in the emergency
identified cause of osteomyelitis in young and/or the clinician should discuss the department, and those who do not
children. This pathogen adds to the case with a pediatric orthopedic surgeon. improve with analgesia should receive an
complexity of interpreting laboratory Any child in which there is concern for appropriate consultation.
studies in limping patients. Clinicians neurovascular injury or compartment Further workup and management
should consider higher level imaging syndrome necessitates an emergent depends on the specific clinical scenario,
when managing a child whose history or surgical consultation. examination findings, laboratory results,
examination is indicative of osteomyelitis, For a child presenting with isolated and radiographs. Such treatments may
regardless of laboratory results. extremity pain or a limp without include arthrocentesis, further imaging
a history of fever or trauma, the (often with MRI), admission for serial
CRITICAL DECISION disposition often depends on the patient’s examinations, or close outpatient follow-
When does a pediatric limp overall appearance and weightbearing up. In many cases, appropriate next
status. If radiographs of the affected steps are not clearly defined or evidence
warrant inpatient admission?
extremity are normal and the patient based; such decisions should be made
The workup and disposition of most is able to bear weight in the emergency collaboratively between the emergency
limping children seen in the acute setting department, primary care follow-up physician, consultants, and the patient
can be determined without consulting a may be appropriate. On the other and/or caregivers.
pediatric orthopedic surgeon. In cases of hand, if an afebrile patient is provided
traumatic injury, most sprains, strains, Summary
NSAID analgesia and remains unable to
buckle fractures, and fractures that are not Most limping has a benign and self-
bear weight, or if the patient’s general
significantly angulated or displaced can be limited etiology; however, emergency
appearance or physical examination is
immobilized with a splint, and the patient clinicians should be mindful of the
concerning, a specialist consultation is
can be sent home with a plan for clinic potentially life- and limb-threatening
necessary.
follow-up. Any affected limb with swelling pathologies that can affect pediatric
The febrile limping child with normal
or tenderness overlying an open physis and patients. Laboratory and radiographic
x-rays, a reassuring white blood cell
normal radiographs should be immobilized studies can be used to help delineate a
count, and normal inflammatory markers,
and treated as an occult (Salter-Harris type diagnosis. Clinicians should consider the
who is able to ambulate in the emergency
I) fracture with clinic follow-up. child’s developmental stage and stated
department following a dose of analgesia,
The patient’s need for follow-up with history, especially when a fracture is
often receives a presumptive diagnosis
a primary care physician or pediatric discovered; nonaccidental trauma may
of transient synovitis. These patients
be present.
orthopedic surgeon should be guided should be advised to continue NSAIDs
In many cases, the cause of a child’s
by the specific injury and local practice and supportive care at home, and should
symptoms may not be readily apparent.
patterns. Fractures that are significantly receive close follow-up care with their
A specialty consultation is warranted
angulated or displaced, as well as intra- primary care provider.
if any laboratory or radiographic
articular fractures, often require reduction Caregivers should be instructed
abnormalities are found. In most cases,
or surgical management. The degree of on appropriate return precautions for
plain radiographs are recommended
as the initial imaging modality.
MRI is an excellent way to evaluate
for osteomyelitis, a septic joint, or
malignancy, but often requires sedation.
Ultrasound can be useful for the detection
of joint effusion. It is not uncommon for
n Dismissing a fracture in a nonambulatory or developmentally delayed child. children to be admitted for observation or
Nonaccidental trauma is an under-recognized and life-threatening diagnosis; further testing. If the patient’s pain cannot
emergency providers should maintain a high index of suspicion when children be reasonably controlled, admission
sustain fractures that do not align with their developmental state or the should be strongly considered.
purported mechanism of injury.
n Assuming a limp is due to a lower-extremity pathology. The back and hip REFERENCES
should not be ignored as possible etiologies of limping. 1. Herman MJ, Martinek M. The limping child. Pediatr
Rev. 2015 May;36(5):184-195.
n Neglecting to order frog-leg views of the hips when assessing for SCFE or 2. Tenenbein M, Reed MH, Black GB. The toddler’s
other hip pathologies. Such findings can be missed on AP views. fracture revisited. Am J Emerg Med. 1990 May;8(3):
208-211.
n Failing to seriously consider more insidious etiologies of pain in a young child 3. Halsey MF, Finzel KC, Carrion WV, Haralabatos SS,
complaining of back pain. Mechanical back pain is very rare in young children. Gruber MA, Meinhard BP. Toddler’s fracture:
presumptive diagnosis and treatment. J Pediatr

May 2018 n Volume 32 Number 5 13


CASE RESOLUTIONS
■ CASE ONE results included a white blood cell 48 hours. Culture results from
count of 22,000, with 90% neutrophils aspirated joint fluid showed
The emergency physician ordered
and 5% lymphocytes. There were no growth of methicillin-sensitive
plain radiographs of the injured
blasts. The child’s ESR and CRP levels Staphylococcus aureus. Pediatric
boy’s right tibia and fibula (AP,
were elevated (90 mm/hr and 7 mg/L, infectious disease specialists were
lateral, and oblique views), which
respectively). Her x-rays were normal. consulted, and a 6-week antibiotic
revealed a clearly visible nondisplaced
Concerned for a septic hip joint or course was initiated, followed by
spiral fracture of the distal tibia,
osteomyelitis, the clinician consulted serial CRP tests.
consistent with a toddler’s fracture.
pediatric orthopedic surgery, which
Although the injury was consistent ■ CASE THREE
recommended inpatient admission.
with accidental trauma, the clinician Although the teenaged soccer
Because the child was not toxic and her
consulted a social worker, who player’s previous imaging reports
irritability resolved, antibiotics were
interviewed the child’s mother were normal, the clinician remained
held. Since she was difficult to examine,
and concurred with the accidental concerned about the patient’s hip.
the orthopedic team suggested an MRI
diagnosis. He was placed in a AP and frog-leg views of the pelvis
of the pelvis with contrast to examine
posterior long-leg splint and pediatric confirmed SCFE on the left with
the femur and joint spaces. The results
orthopedic follow-up care was approximately 15% slippage.
demonstrated enhancement of the
arranged. Pediatric orthopedics was
right femoral head consistent with
■ CASE TWO osteomyelitis. consulted and determined the need
The toddler’s fever, fussiness, Clindamycin and ceftriaxone were for operative repair. The girl was
and limping behavior concerned the initiated, and serial CRP tests were admitted to the hospital and taken to
emergency physician and augmented obtained. After 3 days on intravenous the operating room the next day for
his suspicion about a more serious antibiotics, the child improved and in-situ fixation. She did not require
pathology. The child’s laboratory remained afebrile for the next prophylactic contralateral pinning.

Orthop. 2001 Mar-Apr;21(2):152-156. 18. Gorelick MH, Alpern ER, Alessandrini EA. A system Radiol. 2013 Mar;43 Suppl 1:S8-S22.
4. Livingston KS, Glotzbecker MP, Shore BJ. Pediatric for grouping presenting complaints: the pediatric 31. Gordon JE, Huang M, Dobbs M, Luhmann SJ,
acute compartment syndrome. J Am Acad Orthop emergency reason for visit clusters. Acad Emerg Med. Szymanski DA, Schoenecker PL. Causes of false-
Surg. 2017 May;25(5):358-364. 2005 Aug;12(8):723-731. negative ultrasound scans in the diagnosis of septic
5. Jenny C, Hymel KP, Ritzen A, Reinert SE, Hay TC. 19. Macias CG, Allen CH. Bone, joint and vertebral arthritis of the hip in children. J Pediatr Orthop. 2002
Analysis of missed cases of abusive head trauma. infections. In: Baren JMMMFF, Rothrock SGMFF,
May-Jun;22(3):312-316.
JAMA. 1999 Feb 17;281(7):621-626. Brennan JAMFF, Brown LMMFF, eds. Pediatr Emerg
32. Gaeta M, Minutoli F, Scribano E, et al. CT and MR
6. Dubowitz H, Bennett S. Physical abuse and neglect Med. 2008:705-715.
of children. Lancet. 2007 Jun 2;369(9576):1891-1899. 20. Kimberlin DW, Brady MT, Jackson MA, Long SS, imaging findings in athletes with early tibial stress
7. Diagnostic imaging of child abuse. Pediatrics. 2000 eds. Red Book®: 2015 Report of the Committee on injuries: comparison with bone scintigraphy findings
Jun;105(6):1345-1348. Infectious Diseases. 30th ed. Itasca, IL: American and emphasis on cortical abnormalities. Radiology.
8. Matava MJ, Patton CM, Luhmann S, Gordon JE, Academy of Pediatrics; 2015. 2005 May;235(2):553-561.
Schoenecker PL. Knee pain as the initial symptom 21. Pannaraj PS, Hulten KG, Gonzalez BE, Mason EO 33. White PM, Boyd J, Beattie TF, Hurst M, Hendry GM.
of slipped capital femoral epiphysis: an analysis Jr, Kaplan SL. Infective pyomyositis and myositis Magnetic resonance imaging as the primary imaging
of initial presentation and treatment. J Pediatr in children in the era of community-acquired, modality in children presenting with acute non-
Orthop. 1999 Jul-Aug;19(4):455-460. methicillin-resistant Staphylococcus aureus infection. traumatic hip pain. Emerg Med J. 2001 Jan;18(1):25-29.
9. Murray AW, Wilson NI. Changing incidence of Clin Infect Dis. 2006 Oct 15;43(8):953-960. 34. Averill LW, Hernandez A, Gonzalez L, Peña AH,
slipped capital femoral epiphysis: a relationship 22. Fernandez M, Carrol CL, Baker CJ. Discitis and
Jaramillo D. Diagnosis of osteomyelitis in children:
with obesity? J Bone Joint Surg Br. 2008 vertebral osteomyelitis in children: an 18-year review.
utility of fat-suppressed contrast-enhanced MRI. AJR
Jan;90(1):92-94. Pediatrics. 2000 Jun;105(6):1299-1304.
10. Kelsey JL, Acheson RM, Keggi KJ. The body build 23. de Inocencio J, Carro MÁ, Flores M, Carpio C, Am J Roentgenol. 2009 May;192(5):1232-1238.
of patients with slipped capital femoral epiphysis. Mesa S, Marín M. Epidemiology of musculoskeletal 35. Flynn JM, Widmann RF. The limping child: evaluation
Am J Dis Child. 1972 Aug;124(2):276-281. pain in a pediatric emergency department. and diagnosis. J Am Acad Orthop Surg. 2001 Mar-
11. Siegel DB, Kasser JR, Sponseller P, Gelberman RH. Rheumatol Int. 2016 Jan;36(1):83-89. Apr;9(2):89-98.
Slipped capital femoral epiphysis. A quantitative 24. Frush DP, Frush KS. The ALARA concept in pediatric 36. Cutler L, Molloy A, Dhukuram V, Bass A. Do CT scans
analysis of motion, gait, and femoral remodeling imaging: building bridges between radiology aid assessment of distal tibial physeal fractures?
after in situ fixation. J Bone Joint Surg Am. 1991 and emergency medicine: consensus conference J Bone Joint Surg Br. 2004 Mar;86(2):239-243.
Jun;73(5):659-666. on imaging safety and quality for children in the 37. Connolly LP, Connolly SA, Drubach LA, Jaramillo D,
12. Wenger DR, Ward WT, Herring JA. Legg-Calvé- emergency setting, Feb. 23-24, 2008, Orlando, FL Treves ST. Acute hematogenous osteomyelitis
Perthes disease. J Bone Joint Surg Am. 1991 — Executive Summary. Pediatr Radiol. 2008 Nov;38
of children: assessment of skeletal scintigraphy-
Jun;73(5):778-788. Suppl 4:S629-S632.
based diagnosis in the era of MRI. J Nucl Med. 2002
13. Canavese F, Dimeglio A. Perthes’ disease: 25. Baker N, Woolridge D. Emerging concepts in
prognosis in children under six years of age. J Bone pediatric emergency radiology. Pediatr Clin North Oct;43(10):1310-1316.
Joint Surg Br. 2008 Jul;90(7):940-945. Am. 2013 Oct;60(5):1139-1151. 38. Kocher MS, Zurakowski D, Kasser JR. Differentiating
14. Lee EH, Shafi M, Hui JH. Osteoid osteoma: a 26. Milla SS, Coley BD, Karmazyn B, et al. ACR between septic arthritis and transient synovitis
current review. J Pediatr Orthop. 2006 Sep- Appropriateness Criteria® limping child—ages 0 to 5 of the hip in children: an evidence-based clinical
Oct;26(5):695-700. years. J Am Coll Radiol. 2012 Aug;9(8):545-553. prediction algorithm. J Bone Joint Surg Am. 1999
15. Gereige R, Kumar M. Bone lesions: benign and 27. Baron CM, Seekins J, Hernanz-Schulman M, Yu C, Dec;81(12):1662-1670.
malignant. Pediatr Rev. 2010 Sep;31(9):355-362. Kan JH. Utility of total lower extremity radiography 39. Kocher MS, Mandiga R, Zurakowski D, Barnewolt C,
16. Clarke RT, Van den Bruel A, Bankhead C, Mitchell investigation of nonweight bearing in the young child. Kasser JR. Validation of a clinical prediction rule
CD, Phillips B, Thompson MJ. Clinical presentation Pediatrics. 2008 Apr;121(4):e817-e820. for the differentiation between septic arthritis and
of childhood leukaemia: a systematic review and 28. Hatzenbuehler J, Pulling TJ. Diagnosis and
transient synovitis of the hip in children. J Bone Joint
meta-analysis. Arch Dis Child. 2016 Oct;101(10): management of osteomyelitis. Am Fam Physician.
Surg Am. 2004 Aug;86-A(8):1629-1635.
894-901. 2011 Nov 1;84(9):1027-1033.
17. Jonsson OG, Sartain P, Ducore JM, Buchanan 29. Jarrett DY, Matheney T, Kleinman PK. Imaging SCFE: 40. Caird MS, Flynn JM, Leung YL, Millman JE, D’Italia JG,
GR. Bone pain as an initial symptom of childhood diagnosis, treatment and complications. Pediatr Dormans JP. Factors distinguishing septic arthritis
acute lymphoblastic leukemia: association with Radiol. 2013 Mar;43 Suppl 1:S71-S82. from transient synovitis of the hip in children. A
nearly normal hematologic indexes. J Pediatr. 1990 30. Callahan MJ. Musculoskeletal ultrasonography of prospective study. J Bone Joint Surg Am. 2006
Aug;117(2 Pt 1):233-237. the lower extremities in infants and children. Pediatr Jun;88(6):1251-1257.

14 Critical Decisions in Emergency Medicine


The LLSA
Literature
Review
By Lulu Wang, MD; and Andrew Eyre, MD, MHPEd
Departments of Emergency Medicine at Massachusetts
General Hospital, Brigham and Women’s Hospital, and
Harvard Medical School, Boston, MA
Betz ME, Boudreaux ED. Managing suicidal patients in the emergency department. Ann Emerg Med. 2016 Feb;67(2):276-282.

Managing Suicidal Patients


As many as 8% of all emergency worker). From there, inpatient admission course of emergency department
department patients report recent may be warranted. treatment, and providing basic comforts.
suicidal ideations. Clinicians can In the absence of a specialist, an Attempts at verbal de-escalation should
use a suicide risk stratification tool to emergency provider can perform a more be made before resorting to chemical or
identify those who are most vulnerable. thorough assessment using a tool such physical restraints. High-risk patients
The intent to self-harm is not always as the Suicide Assessment Five-Step should be protected from self-harm;
readily admitted, however, so patients Evaluation and Triage (SAFE-T) protocol. dangerous objects should be removed;
at risk should be appropriately This includes, but is not limited to, the and the patient should be observed closely
screened. The evaluation should be identification of risk factors (psychiatric and prevented from leaving without
conducted only in those who are sober history, prior suicide attempts, substance psychiatric clearance. As with any case,
and have a capacity for decision- use, family history, stressors), protective a focused medical evaluation should be
making. Preliminary risk stratification factors, suicide inquiry (ideation, plan, performed to assess for problems that
contains questions about plan, intent, intent), determination of risk level (low may be contributing to the patient’s
past attempts, mental health history, risk can be addressed on an outpatient psychiatric complaints.
substance abuse, and issues with basis; high risk requires psychiatric
irritability or agitation. Example assessment and likely admission), and
questions include: documentation. KEY POINTS
n Have you been thinking about how A number of quick, directed n Not every patient with suicidal
you might kill yourself recently? interventions can be quite impactful. ideations requires a comprehen­
n Do you intend to kill yourself? These include patient education sive psychiatric consultation in
n Have you ever tried to kill yourself? (instilling hope, reducing shame and the emergency department.
n Have you ever been treated for stigma, providing clear follow-up and The proper use of validated
mental health problems? return instructions), safety planning screening and risk stratification
n Has drinking or drug use been a (creating a structured plan with the tools, including the SAFE-T, can
problem for you? patient, including coping strategies help identify the small subset of
n Have you been feeling anxious or and people to contact), lethal means patients at high risk for suicide.
agitated recently? counseling (reducing access to guns, n Patients with suicidal thoughts
Patients who answer “no” to all toxic medications, etc.), rapid referral represent an extremely vulnerable
of the above questions may not need population that benefits from a
(arranging for follow-up within 7 days),
therapeutic alliance, empathy, and
further psychiatric assessment and can and identifying a social safety net
open communication.
be discharged with a careful outpatient (communicating with family and other
n High-risk signs include suicidal
follow-up plan. A “yes” answer to providers regarding the visit).
intention, having a plan, previous
any of the above questions warrants Finally, it is crucial to establish a
attempts, alcohol or drug use,
a comprehensive risk assessment thera­peutic alliance. This can be done
psychiatric history, and recent
with a mental health specialist (eg, by communicating directly, expressing
anxiety, irritability, or aggression.
psychiatrist, psychologist, or social empathy, explaining the anticipated

Critical Decisions in Emergency Medicine’s series of LLSA reviews features articles from ABEM’s 2018 Lifelong Learning and
Self-Assessment Reading List. Available online at acep.org/llsa and on the ABEM website.

May 2018 n Volume 32 Number 5 15


The Critical Image
A 74-year-old woman presents with right hip pain that has prevented By Joshua S. Broder, MD, FACEP
her from walking for several weeks. She says her symptoms began Dr. Broder is an associate professor and the
2 months earlier, after she fell out of bed. Her daughters report that the residency program director in the Division of
Emergency Medicine at Duke University Medical
patient complained of some hip pain preceding her fall; in fact, she visited Center in Durham, North Carolina.
an orthopedist 1 year earlier for the same complaint. The patient also notes
Case contributor: Lindsey Rowe, MHS, PA-C
some weight loss.
Her vital signs are blood pressure 162/71, heart rate 67, temperature 36.9°C (98.4°F), respiratory rate 16, and oxygen saturation
100% on room air. The patient appears to be in no distress. Although she complains of severe right hip pain with flexion, the limb
is normal in length compared with the unaffected left side and has normal pulses. Her distal sensation and motor examinations
are normal. Suspecting a hip fracture, the emergency physician orders radiographs of the right hip and pelvis and compares
these with prior images. Following x-rays, a computed tomography scan is also obtained.

A. Radiograph of the pelvis


A
from 1 year earlier. The pelvic
Bowel gas
Bowel ring is intact, and both femoral
gas necks are normal, without
signs of fracture. The femoral
heads have a normal, rounded
appearance, without significant
evidence of degenerative
Normal changes. Scattered bowel
femoral neck gas creates a rounded lucent
appearance over the pelvis.

The normal pelvis


forms a complete
ring

B. Radiograph of the pelvis B


on the date of the emergency
department visit. The femoral
necks are intact. A lucent area
over the right iliac wing resembles Lytic
bowel gas; however, it interrupts lesion
Intact iliac
the normal ring of the pelvis, of iliac
bone
confirming the presence of a bone
lytic lesion. Lytic lesion
interrupts
pelvic ring

16 Critical Decisions in Emergency Medicine


C. CT with
C
KEY POINTS
intravenous
n Following a low-mechanism fall from
contrast,
standing, hip injuries such as femoral
soft-tissue
neck and intertrochanteric fractures
windows.
This image are common in older patients. These
shows a high-risk areas should be inspected
soft-tissue closely, but in addition, x-rays of the
mass pelvis should be read systematically,
replacing with attention paid to the bony ring of
the right the pelvis. Disruption of this ring can
iliac bone. indicate fracture, or — as in this case
Soft tissue
— a destructive lytic lesion. Lucencies
mass
replacing from bowel gas can simulate lytic
iliac bone findings, but do not usually interrupt
the pelvic ring.
n When previous images are available,
D. CT with comparison is useful; bowel gas
intravenous D
will move over time, and other new
contrast, findings may be evident. When
bone uncertainty exists, CT or magnetic
windows.
resonance imaging can provide
Again, a
additional information.
lytic lesion
of the right
iliac bone is
seen.
CASE RESOLUTION
The patient underwent a CT-
Lytic lesion Normal
iliac bone guided needle biopsy, which
of iliac
bone
demonstrated a diffuse large
B-cell lymphoma.

E F Lytic
lesion
Normal of iliac
iliac bone
bone

Lytic
lesion
of iliac
bone

E. 3D CT reconstruction, frontal view of pelvis. The lesion is F. 3D CT reconstruction, posterior view of pelvis. From this
seen as a small erosion in the iliac bone. vantage point, the much larger destructive extent of the lesion
is seen.

May 2018 n Volume 32 Number 5 17


The Critical
Helix

Triangular fossa

Procedure
Scapha Ant
i he lix

Concha

Tragus
He
lix

Intertragical By Harish Kotipoyina, MD; and Steven Warrington, MD, MEd, FACEP
notch Dr. Kotipoyina is a family medicine resident at North Florida Regional Hospital in Gainesville,
FL. Dr. Warrington is director of the Emergency Medicine Residency Program and academic
Antitragus chair of the Department of Emergency Medicine at Orange Park Medical Center in Orange
Lobule Park, FL.

AURICULAR HEMATOMA INCISION AND DRAINAGE


Auricular/perichondrial hematomas generally occur when the auricle of the ear suffers blunt
trauma, leading to the tearing of blood vessels. The bleeding can cause a hematoma to form in the
subperichondrial space. Such injuries require early management to prevent permanent deformity.

Benefits and Risks One alternative to incision and TECHNIQUE


Although incision and drainage can drainage of the hematoma is needle
1. Gather supplies: No. 15 scalpel,
prevent permanent cosmetic deformities aspiration; however, this approach is
5-mL syringe with a 23- or 27-gauge
of the ear and further damage to the associated with a higher recurrence
needle for anesthesia, 1% lidocaine
cartilage, hematomas older than 7 days rate. When using this route, the patient
(with or without epinephrine),
require ENT management. Risks of should be aware of the potential need for
irrigation setup (if using), suction (if
the technique include infection repeated drainage.
using), scissors, iodine or ChloraPrep,
(perichondritis), scar formation, procedure
Reducing Side Effects sutures (4-0 vicryl), suturing kit,
failure, and hematoma recurrence. The splint material, gloves, and eye
An auricular block can provide
risk of a poor cosmetic outcome also protection.
excellent anesthesia without creating
should be discussed with the patient. 2. Place the patient in the supine or
further difficulties when re-approxi­
Alternatives mating the anatomy. lateral decubitus position on the
Various materials may be used to While there is no clear evidence unaffected side.
create a pressure dressing, which acts as regar­ding the efficacy of antibiotics 3. Clean, prep, and anesthetize the area;
a splint/bolster and prevents hematoma in such cases, prophylactic antibiotics consider an auricular block.
accumulation. Although the author may be considered when managing 4. Make a small incision on the edge of
certain patients, including those who are the base of the hematoma, along the
recommends rolled gauze due to its ready
immunocompromised. Pseudomonas natural curve of the skinfold.
accessibility, other alternatives include
5. Suction the hematoma, or use direct
cotton, plastic bolsters, silicone splints, aeruginosa and Staphylococcus aureus
pressure to express it out.
and dental kit materials. Several of these are among the most common offending
6. Consider saline irrigation.
items may offer better support and lead to organisms.
7. Reapproximate the perichondrium
improved cosmetic outcomes. The ear requires time to heal. During
with the cartilage, and suture the
Some clinicians opt for a tie-over the first week, patients should refrain
incision closed.
dressing that involves a through-and- from risky activities and encouraged to
8. Apply the compression dressing/splint
through stitch of the antihelix or helix. wear headgear to protect against future
and secure. This can be achieved with
The suture can help position the splint injury.
a through-and-through suture or with
material (gauze) snug to the ear, enabling It may be beneficial to re-evaluate
a circum­ferential head wrap. Note
better compression. In addition, the the patient every 24 hours for 3 to 5
that the dressing should not be so tight
punctures caused by the stitch may allow days to monitor for the reaccumulation
as to cause vascular compromise.
further drainage of the hematoma and help of blood and signs of infection. If 9. Consider prophylactic antibiotics,
prevent recurrence. It should be noted, possible, antiplatelet and anticoagulant and counsel the patient on follow-up
however, that such dressings can increase medications should be avoided while the and return precautions.
the risk of a poor cosmetic outcome. injury heals.

18 Critical Decisions in Emergency Medicine


Bitter Pill
Adverse Drug Events
in Geriatric Patients

LESSON 10

By Thuy Van Pham, MD


Dr. Pham is an attending emergency physician at Good Samaritan Hospital in
Puyallup, Washington.
Reviewed by Steven J. Warrington, MD, MEd

OBJECTIVES
On completion of this lesson, you should be able to: CRITICAL DECISIONS
1. Understand the physiological changes that arise with
n Which age-related physiological changes affect
aging that affect how the body absorbs, metabolizes,
how drugs are absorbed, metabolized, and
and eliminates drugs.
eliminated from the body?
2. Identify circumstances that contribute to polypharmacy.
n What circumstances contribute to polypharmacy,
3. Define adverse drug events (ADEs) and recognize them
in the emergency department. and what can you do to help prevent it?

4. Describe the drugs most commonly implicated in ADEs n How can adverse drug events be recognized and
in geriatric patients. managed in the emergency department?
5. Identify ways to prevent ADEs and educate elderly n What medications should raise suspicion for
patients about medication compliance.
adverse drug events in geriatric patients?

FROM THE EM MODEL n What steps can be taken to prevent adverse drug
17.0 Toxicologic Disorders events?
17.1 Drug and Chemical Classes

Senior citizens, who constitute one of the fastest growing segments of our society, are visiting emergency
departments more frequently for drug-related complications. By understanding the unique physiological
factors that affect geriatric patients, however, emergency clinicians can minimize the risk of these potentially deadly
complications.

May 2018 n Volume 32 Number 5 19


CASE PRESENTATIONS
■ CASE ONE ■ CASE TWO are hemoglobin 12 g/dL, hematocrit
A 76-year-old Caucasian woman A 68-year-old African American 36%, sodium 138 mmol/L, potassium
presents with progressively worsening man arrives via ambulance for altered 5.2 mmol/L, BUN 60 mg/dL, and
lower abdominal and back pain. She mental status. His family called 911 creatinine 1.7 mg/dL.
describes the discomfort as an ache, when he began slurring his words.
and nothing seems to make it better EMS found him to be awake and alert ■ CASE THREE
or worse. She denies any trauma or but disoriented to self, place, and time, An 85-year-old woman presents
injury. She complains of nausea and with no obvious neurological deficits. with epigastric abdominal pain, She
cold sweats, but has no fever, chills, or His initial vital signs were temperature explains that her symptoms began
vomiting. 37°C (98.6°F), blood pressure 145/65, about a week ago, but are much worse
The patient’s medical history pulse rate 84, respiratory rate 14, and today. The pain is exacerbated by
includes hypertension, hyperlipidemia, fingerstick glucose 40. EMS also noted movements and taking deep breaths.
and atrial fibrillation. Her that the patient is hypoglycemic and
She has been taking ibuprofen with
medications include metoprolol, gave him 50 mL of dextrose 50%.
no significant relief. She denies any
hydrochlorothiazide, atorvastatin, His medical history includes
dysuria, but states that her stool has
and warfarin. Her last international hypertension, diabetes, chronic venous
normalized ratio (INR) was checked stasis, chronic kidney disease, high been darker for weeks.
2 weeks ago and is reported as 2.1. cholesterol, a prior cerebral vascular Her medical history includes
She was recently diagnosed with a accident with no residual deficits, and hypertension, high cholesterol, and
urinary tract infection (UTI) by her coronary artery disease. He takes osteoarthritis. She takes amlodipine,
primary care physician and is on Day lisinopril, glipizide, simvastatin, meloxicam, atorvastatin, and a
3 of 5 of ciprofloxacin. She has had no metoprolol, clopidogrel, and 81 mg multivitamin daily. She is otherwise
prior surgeries. of aspirin. He started taking healthy and independent and lives in
The patient is alert, awake, and cephalexin and sulfamethoxazole/ a senior living facility. Her vital signs
oriented; she is uncomfortable but not trimethoprim (SMZ/TMP) 2 days ago
are temperature 37.3°C (99.1°F), blood
in any acute distress. Her vital signs for uncomplicated lower-extremity
pressure 170/100, pulse rate 110,
are temperature 37.9°C (100.2°F), cellulitis.
respiratory rate 16, and fingerstick
blood pressure 102/67, pulse rate On arrival, he is alert, awake,
115, and respiratory rate 17. Her and oriented to self. He denies any glucose 102. She appears clammy and
examination is only significant for pain. The patient’s repeat vital signs is tripoding on the bed.
a diffusely tender abdomen without at triage are temperature 36.4°C The patient’s lung and cardiac
evidence of guarding, rebound, or (97.5°F), blood pressure 162/75, examinations are unremarkable, other
rigidity. pulse rate 98, respiratory rate 20, and than tachycardia. Her abdomen is soft,
She has no costovertebral angle fingerstick glucose 77. His neurological but is diffusely tender to palpation.
tenderness or midline tenderness to examination is normal, and he has Her stool is tarry black and guaiac
palpation of her back. She is given 1 L no facial droop, slurring of speech, positive, but otherwise unremarkable.
of normal saline. Her blood test results or asymmetry. His lower extremities Her blood test results are hemoglobin
are hemoglobin 9.3 g/dL, hematocrit show chronic venous stasis with pitting
7.8 g/dL, hematocrit 22%, platelet
28%, sodium 138 mmol/L, potassium edema up to the knees bilaterally,
200, sodium 142 mmol/L, potassium
4.5 mmol/L, bilirubin 1.5, BUN with overlying erythema that is
60 mg/dL, and creatinine 1.2 mg/dL. nonpurulent. The erythema appears 4.5 mmol/L, bilirubin 1.5, BUN
In the emergency department, her chronic in nature, and there is obvious 60 mg/dL, creatinine 1.1 mg/dL,
INR is 8.5, and an ECG shows normal hyperpigmentation and thickening of aspartate aminotransferase (AST) 55,
sinus tachycardia. A CT scan of her the skin. Otherwise, his examination alanine transaminase (ALT) 58, and
abdomen is ordered. is unremarkable. His blood test results alkaline phosphatase 130.

According to the US Census Bureau, comorbidities for which they are taking of these patients take at least one
adults 65 years and older comprise multiple medications.2 This population over-the-counter (OTC) medication.4
13% of the country’s population.1-3 By consumes 32% of all prescription Unfortunately, indiscriminate
the year 2030, the number of elderly drugs and contributes to 35% of the prescribing practices often lead to
Americans is expected to grow to nation’s total drug expenditure.4 More an overlap of prescription and OTC
20%.1,2 Geriatric patients, who are than 90% of elderly patients use at medications, which increases the risk
particularly complex to manage in the least one medication; on average, they of drug-drug interactions and adverse
acute setting, often suffer from multiple use four drugs per person.4 Two-thirds drug events (ADEs).5,6

20 Critical Decisions in Emergency Medicine


CRITICAL DECISION decreases, as does bone mass and total an increased elimination half-life
body water.2 While total body fat and a prolonged effect.8 Examples of
Which age-related physiological
increases, total body weight decreases.4 common fat-soluble agents include
changes affect how drugs are benzodiazepine and phenytoin.8
Age-related changes in gastric absorption
absorbed, metabolized, and increase gastric pH, decrease motility, The aging process also causes an
eliminated from the body? and reduce splanchnic blood flow, all overall decrease in serum albumin. This
Aging is a natural inevitability of which affect drug absorption rates.8 change affects medications that are
associated with an increased risk of Natural declines in kidney and liver bound to the protein, increasing the free
morbidity and mortality; however, it function also affect the body’s ability to drug concentration and thus the risk of
is not a simple process generalizable process medications. toxicity, even at recommended dosing.
Pharmacokinetics, the means Examples of drugs that are albumin-
to everyone. The geriatric population
by which drugs are metabolized, is bound include warfarin, phenytoin, and
is a heterogeneous group of people
affected by a patient’s ability to absorb, salicylic acid.4,8
with substantial varia­bility in health,
distribute, and eliminate them.8 The The liver is the primary metabolizer
disability, and comorbidities.4,5,7
distribution of drugs in elderly patients of most medications, while the kidneys
While one’s biological age is
is particularly affected by decreased are responsible for the majority of drug
influenced by specific factors such as
excretion and elimination. After 40 years
comorbidities, genetics, and social lean body weight and an increase in
of age, glomerular function falls 6%
circumstances, chronological aging central adipose tissue.8 Water-soluble
to 10% per decade. Tubular function
coincides with important — and agents such as digoxin and alcohol
is reduced 30% to 40% by the time a
universal — physiological changes have a reduced volume of distribution,
patient reaches 90 years of age. Keeping
(Figure 1). Some of these changes which causes an increase in the initial
these changes in mind, the dosage of
affect how medications are absorbed, drug concentration.4,8 Fat-soluble drugs,
drugs that are excreted by filtration in
metabolized, and eliminated from the on the other hand, have an increased
the kidneys or renal tubules may need to
body: As people age, lean body weight volume of distribution, leading to
be reduced.4,8
Pharmacodynamics, the way drugs
affect the body, encompasses complex
FIGURE 1. Physiological Changes Associated with Aging
processes that are not well understood.
Changes in drug response with aging
are assumed, but have not been well
explained in the literature. An elderly
patient may demonstrate an inadequate
response to certain medications at
Liver function tests regular doses, yet be overly sensitive
Gastric pH
Liver mass to others. For example, it is well
H. pylori infection
Overall liver function documented that elderly people have
Peristalsis/gastric motility reduced inotropic and chronotropic
Hepatic cell mass and responses to β1-adrenergic stimulation
Total body fat function but an increased sensitivity to drugs
(adipose) such as warfarin, benzodiazepines, and
opiates.8
Total body weight Prostatic
CRITICAL DECISION
Lean body weight hypertrophy (men)
Bone mass What circumstances contribute
Pelvic floor weakness
Total body water (women)
to polypharmacy, and what can
you do to help prevent it?
Glomerular filtration Geriatric patients tend to seek
rate help from an array of practitioners,
which can lead to polypharmacy,
Urine concentration
in which a single patient with one
capacity
or more conditions uses multiple
drugs simultaneously. Treatment by
multiple practitioners often leads
to uncoordinated care, overlapping
prescriptions and, ultimately, an
increased risk of ADEs. “Prescribing

May 2018 n Volume 32 Number 5 21


cascades” are all too common, with Additionally, many pills look of an ADE is 35%; when a patient is
multiple providers prescribing various similar, which can lead to confusion. taking seven or more medications, the
medications for several comorbidities As these situations arise, clinicians risk increases to 82%.6,13
while having little or no communication should consider ordering medications in ADEs can be vague and nonspecific,
with each other.5 Moreover, duplicates bottles that can be opened easily by the so it is important for emergency
within the same class of drugs may be patient and should consider requesting clinicians to keep this diagnosis in
prescribed and/or drug-drug interactions larger print on labels. The incidence mind when managing any geriatric
may go unrecognized, leading to of mistakes increases 15-fold when the patient. Take an additional few minutes
treatment with yet more drugs.6 number of prescriptions increases from to ask about any medication-related
The inappropriate use of medication one drug to four.9 habits and/or changes, and to review
and noncompliance with dosage the patient’s current prescription
CRITICAL DECISION list, if available. If you have access
instructions contribute to polypharmacy
and concomitant ADEs.4 Nonadherence, How can adverse drug events be to pharmacists, request medication
most commonly in the form of omission recognized and managed in the reconciliations for your geriatric
(seen in ≤90% of cases), can be either emergency department? patients.
intentional or unintentional. Patients
often have little understanding of
ADEs include any injury related to CRITICAL DECISION
medication use. They can be caused
their medications and the indications What medications should raise
by the side effects or interactions of
for each, and may not take them as
medications. These reactions constitute suspicion for adverse drug
instructed. One-fourth to one-half of events in geriatric patients?
the fourth to sixth greatest causes of
elderly patients do not adhere to the death in the US and contribute to 5% Several guidelines have been
medication instructions they receive; to 10% of all hospital admissions.7,11 published to help physicians avoid
about two-thirds of this noncompliance Approximately 10% to 20% of all prescribing dangerous drugs to elderly
is intentional, resulting from the desire hospital admissions involving the elderly patients. The best-known publication is
to avoid side effects they experience stem from an iatrogenic cause, the the American Geriatrics Society 2015
with the full dose, an attempt to boost most common being an ADE from a Updated Beers Criteria for Potentially
the therapeutic response, or concern cardiovascular or psychotropic drug.10 Inappropriate Medication Use in Older
about the high cost of the medication.4 In elderly patients, these events Adults (also known as the Beers List),
Moreover, patients might take an are more likely to be under- which was first published in 1991. The
incorrect dose, hoard drugs for future reported, severe, and less likely to be list, which was most recently revised in
use, or self-prescribe OTC and herbal recognized.7,12,13 Older patients are 2015 (Table 1), includes medications
medications.8 seven times more likely than younger that should be avoided in nursing homes
Health care providers should consider patients to experience an ADE requiring because of their potential danger to
factors such as the patient’s literacy level; hospitalization.14,15 Approximately half geriatric patients.16 It was compiled
the presence of arthritic pain, which can of these cases occur in patients older using data from common drugs taken
make it difficult to open pill bottles; the than 80 years; ADEs are more than by office-based clinic patients, nursing
tendency of some patients to transfer three times as common in those older home patients, and community-dwelling
pills to nonoriginal bottles or to combine than 90 years.12 The more medications elderly individuals who experienced
pills into fewer bottles; and diminished a patient takes, the greater the risk of a possible ADEs.17 Examples of side effects
eyesight, which can hamper the ability to drug-drug or drug-disease interaction. range from hypotension to dizziness
read and comprehend labels.8 When two drugs are prescribed, the risk to altered mental status from central
nervous system suppression. However,
this study did not include drugs that are
prescribed in acute care settings.17
Although the Beers List has been
somewhat useful in guiding emergency
department practice, approximately
n Though aging is a process that affects everyone differently, consider how 90% of ADEs in acute care settings stem
common physiological changes may affect the absorption, metabolism, and from medications excluded from the
elimination of drugs on a patient’s current medication list. list.15,17 Nine out of 10 acute visits for
n Polypharmacy is common; the risk of ADEs increases with every additional ADEs are related to oral anticoagulants/
medication. antiplatelets, diabetic medications,
n Consider ADEs when assessing any patient complaint, especially in the elderly or drugs with a narrow therapeutic
population. window.15
n The most common drugs involved in ADEs in geriatric patients include
Three medications — warfarin,
anticoagulants, diabetic medications, and digoxin.
insulin, and digoxin — are the most

22 Critical Decisions in Emergency Medicine


TABLE 1. 2015 American Geriatrics Society (AGS) Beers Criteria for Potentially Inappropriate Medication Use16
Strength of
Organ System, Therapeutic Quality of Recommen­
Category, Drug Rationale Recommendation Evidence dation
ANTICHOLINERGICS
First-generation antihistamines Highly anticholinergic; clearance reduced with Avoid Moderate Strong
Brompheniramine advanced age, and tolerance develops when used as
Carbinoxamine a hypnotic; risk of confusion, dry mouth, constipation,
Chlorpheniramine and other anticholinergic effects or toxicity
Clemastine
Cyproheptadine
Dexbrompheniramine
Dexchlorpheniramine
Dimenhydrinate
Diphenhydramine (oral) Diphenhydramine may be appropriate in some
Doxylamine situations (eg, for the acute treatment of severe
Hydroxyzine allergic reactions)
Meclizine
Promethazine
Triprolidine
Antiparkinsonian agents Not recommended for prevention of extrapyramidal Avoid Moderate Strong
Benztropine (oral) symptoms with antipsychotics; more-effective agents
Trihexyphenidyl available for treatment of Parkinson disease
Antispasmodics Highly anticholinergic; uncertain effectiveness Avoid Moderate Strong
Atropine (excludes ophthalmic)
Belladonna alkaloids
Clidinium-chlordiazepoxide
Dicyclomine
Hyoscyamine
Propantheline
Scopolamine
ANTITHROMBOTICS
Dipyridamole (oral short-acting) May cause orthostatic hypotension; more effective Avoid Moderate Strong
(does not apply to the extended-release alternatives available; intravenous form acceptable
combination with aspirin) for use in cardiac stress testing
Ticlopidine Safer, effective alternatives available Avoid Moderate Strong
ANTI-INFECTIVE
Nitrofurantoin Potential for pulmonary toxicity, hepatotoxicity, Avoid in individuals with Low Strong
and peripheral neuropathy, especially with long-term creatinine clearance <30
use; safer alternatives available mL/min or for long-term
suppression of bacteria
CARDIOVASCULAR
Peripheral alpha-1 blockers High risk of orthostatic hypotension; not Avoid use as an Moderate Strong
Doxazosin Prazosin recommended as routine treatment for hyper­tension; antihypertensive
Terazosin alternative agents have superior risk-benefit profile
Central alpha blockers High risk of adverse CNS effects; may cause Avoid clonidine as first- Low Strong
Clonidine Guanabenz bradycardia and orthostatic hypotension; not antihypertensive; avoid
Guanfacine Methyldopa recommended as routine treatment for others as listed
Reserpine (>0.1 mg/d) hypertension
Disopyramide Disopyramide is a potent negative inotrope and Avoid Low Strong
therefore may induce heart failure in older adults;
strongly anticholinergic; other antiarrhythmic drugs
preferred
Dronedarone Worse outcomes have been reported in patients Avoid in individuals High Strong
who have permanent atrial fibrillation or severe or with permanent atrial
recently decompensated heart failure fibrillation or severe or
recently decompensated
heart failure

May 2018 n Volume 32 Number 5 23


TABLE 1. 2015 American Geriatrics Society (AGS) Beers Criteria (continued)
Strength of
Organ System, Therapeutic Quality of Recommen­
Category, Drug Rationale Recommendation Evidence dation
Digoxin Use in atrial fibrillation: should not be used as a first- Avoid as first-line therapy Moderate Strong
line agent in atrial fibrillation because more-effective for atrial fibrillation
alternatives exist and it may be associated with
increased mortality
Use in heart failure: questionable effects on risk of Avoid as first-line therapy Low Strong
hospitalization and may be associated with increased for heart failure
mortality in older adults with heart failure; in heart
failure, higher dosages are not associated with
additional benefit and may increase risk of toxicity
Decreased renal clearance of digoxin may lead to If used for atrial Dosage Dosage
increased risk of toxic effects; further dose reduction fibrillation or heart >0.125 mg/d: >0.125 mg/d:
may be necessary in patients with Stage 4 or 5 chronic failure, avoid dosages Moderate Strong
kidney disease >0.125 mg/d
Nifedipine (immediate release) Potential for hypotension; risk of precipitating Avoid High Strong
myocardial ischemia
Amiodarone Amiodarone is effective for maintaining sinus rhythm Avoid amiodarone as High Strong
but has greater toxicities than other antiarrhythmics first-line therapy for atrial
used in atrial fibrillation; it may be reasonable first-line fibrillation unless patient
therapy in patients with concomitant heart failure has heart failure or
or substantial left ventricular hypertrophy if rhythm substantial left ventricular
control is preferred over rate control hypertrophy
CENTRAL NERVOUS SYSTEM
Antidepressants Highly anticholinergic, sedating, and cause Avoid High Strong
(alone or in combination) orthostatic hypotension; safety profile of low dose
Amitriptyline Amoxapine doxepin (6 mg/d) comparable with that of placebo
Clomipramine Desipramine
Doxepin (>6 mg/d) Imipramine
Nortriptyline Paroxetine
Protriptyline Trimipramine
Antipsychotics Increased risk of cerebrovascular accident (stroke) Avoid, except for Moderate Strong
(first- and second-generation) and greater rate of cognitive decline and mortality schizophrenia, bipolar
in persons with dementia. Avoid antipsychotics disorder, or short-term
for behavioral problems (eg, dementia, delirium) use as antiemetic during
unless nonpharmacological options (eg, behavioral chemotherapy
interventions) have failed or are not possible and
the patient is threatening substantial harm to self
or others
Barbiturates High rate of physical dependence and tolerance Avoid High Strong
Amobarbital Butabarbital to sleep benefits; greater risk of overdose at low
Butalbital Mephobarbital dosages
Pentobarbital Phenobarbital
Secobarbital
Benzodiazepines Older adults have increased sensitivity to Avoid Moderate Strong
(short- and intermediate-acting) benzodiazepines and decreased metabolism of long-
Alprazolam Estazolam acting agents; in general, these drugs increase risk of
Lorazepam Oxazepam cognitive impairment, delirium, falls, fractures, and
Temazepam Triazolam motor vehicle crashes in older adults
Benzodiazepines (long-acting) Avoid Moderate Strong
Clorazepate Clonazepam
Diazepam Flurazepam
Quazepam
Chlordiazepoxide Chlordiazepoxide may be appropriate for seizure
(alone or in combination with disorders, rapid eye movement sleep disorders,
amitriptyline or clidinium) benzodiazepine withdrawal, ethanol withdrawal,
severe generalized anxiety disorder, and
periprocedural anesthesia

24 Critical Decisions in Emergency Medicine


TABLE 1. 2015 American Geriatrics Society (AGS) Beers Criteria (continued)
Strength of
Organ System, Therapeutic Quality of Recommen­
Category, Drug Rationale Recommendation Evidence dation
Meprobamate High rate of physical dependence; very sedating Avoid Moderate Strong
Nonbenzodiazepine, benzodiazepine Benzodiazepine receptor agonists have adverse Avoid Moderate Strong
receptor agonists effects similar to those of benzodiazepines in older
Eszopiclone adults (eg, delirium, falls, fractures); increased
Zolpidem emergency department visits and hospitalizations;
Zaleplon motor vehicle crashes; minimal improvement in sleep
latency and duration
Ergoloid mesylates Lack of efficacy Avoid High Strong
(dehydrogenated ergot alkaloids)
Isoxsuprine
ENDOCRINE
Androgens Potential for cardiac problems; contraindicated in Avoid unless indicated Moderate Weak
Methyltestosterone men with prostate cancer for confirmed
Testosterone hypogonadism with
clinical symptoms
Desiccated thyroid Concerns about cardiac effects; safer alternatives Avoid Low Strong
available
Estrogens (with or without progestins) Evidence of carcinogenic potential (breast and Avoid oral and topical Oral and patch: Oral and
endometrium); lack of cardioprotective effect patch; acceptable to use High patch:
and cognitive protection in older women. Vaginal low-dose intravaginal Strong
estrogens for the treatment of vaginal dryness are estrogen (eg, cream or
safe and effective; women with a history of breast tablets) for management Vaginal cream Topical
cancer who do not respond to nonhormonal therapies of dyspareunia, lower or tablets: vaginal
are advised to discuss the risk and benefits of low- urinary tract infections, Moderate cream or
dose vaginal estrogen (estradiol <25 µg twice weekly) and other vaginal tablets:
with their health care provider symptoms Weak
Growth hormone Impact on body composition is small and associated Avoid, except as High Strong
with edema, arthralgia, carpal tunnel syndrome, hormone replacement
gynecomastia, and impaired fasting glucose after pituitary gland
removal
Insulin (sliding scale) Higher risk of hypoglycemia without improvement Avoid Moderate Strong
in hyperglycemia management, regardless of care
setting; refers to sole use of short- or rapid-acting
insulins to manage or avoid hyperglycemia in absence
of basal or long-acting insulin; does not apply to
titration of basal insulin or use of additional short- or
rapid-acting insulin in conjunction with scheduled
insulin (ie, correction insulin)
Megestrol Minimal effect on weight; increases risk of thrombotic Avoid Moderate Strong
events and possibly death in older adults
Sulfonylureas (long-duration) Avoid High Strong
Chlorpropamide Chlorpropamide has a prolonged half-life in older
adults; can cause prolonged hypoglycemia; causes
syndrome of inappropriate antidiuretic hormone
secretion

Glyburide Glyburide poses a higher risk of severe prolonged


hypoglycemia in older adults
GASTROINTESTINAL
Metoclopramide Can cause extrapyramidal effects, including tardive Avoid, unless for Moderate Strong
dyskinesia; risk may be greater in frail older adults gastroparesis
Mineral oil (given orally) Potential for aspiration and adverse effects; safer Avoid Moderate Strong
alternatives are available

May 2018 n Volume 32 Number 5 25


TABLE 1. 2015 American Geriatrics Society (AGS) Beers Criteria (continued)
Strength of
Organ System, Therapeutic Quality of Recommen­
Category, Drug Rationale Recommendation Evidence dation
Proton-pump inhibitors Risk of Clostridium difficile infection and bone loss Avoid scheduled use for High Strong
and fractures >8 weeks unless for high-
risk patients (eg, oral
corticosteroids or chronic
NSAID use), erosive
esophagitis, Barrett’s
esophagitis, pathological
hypersecretory
condition, or
demonstra­ted need for
maintenance treatment
(eg, due to failure of drug
discontinuation trial or
H2 blockers)

PAIN MEDICATIONS

Meperidine Not effective in dosages commonly used; may pose Avoid, especially in Moderate Strong
higher risk of neurotoxicity, including delirium, than patients with chronic
other opioids; safer alternatives exist kidney disease

Non-cyclooxygenase-selective NSAIDs Increased risk of gastrointestinal bleeding or peptic Avoid chronic use, unless Moderate Strong
(oral) ulcer disease in high-risk groups, including those other alternatives are not
Aspirin (>325 mg/d) >75 years or taking oral or parenteral corticosteroids, effective and patient can
Diclofenac Diflunisal anticoagulants, or antiplatelet agents; use of proton- take gastroprotective
Etodolac Fenoprofen pump inhibitor or misoprostol reduces but does not agent (proton-pump
Ibuprofen Ketoprofen eliminate risk. Upper gastrointestinal ulcers, gross inhibitor or misoprostol)
Meclofenamate Mefenamic acid bleeding, or perforation caused by NSAIDs occur in
Meloxicam Nabumetone approximately 1% of patients treated for 3–6 months
Naproxen Oxaprozin and in ~2%–4% of patients treated for 1 year; these
Piroxicam Sulindac trends continue with longer duration of use
Tolmetin

Indomethacin Of all the NSAIDs, indomethacin has the most Avoid Moderate Strong
adverse effects

Ketorolac (includes parenteral) Ketorolac increases risk of GI bleeding, peptic ulcer Avoid Moderate Strong
disease, and acute kidney injury in elderly patients

Pentazocine Causes CNS adverse effects, including confusion Avoid Low Strong
and hallucinations, more commonly than other
opioid analgesic drugs; is also a mixed agonist and
antagonist; safer alternatives exist

Skeletal muscle relaxants Most muscle relaxants are poorly tolerated by Avoid Moderate Strong
Carisoprodol Chlorzoxazone geriatric patients due to drug-related complications
Cyclobenzaprine Metaxalone (eg, anticholinergic adverse effects, sedation,
Methocarbamol Orphenadrine increased risk of fractures); effectiveness at dosages
tolerated by older adults is questionable

GENITOURINARY

Desmopressin High risk of hyponatremia; safer alternatives exist Avoid for treatment of Moderate Strong
nocturia or nocturnal
polyuria

The intentions of the criteria are to improve the selection of prescription drugs by clinicians and patients; evaluate patterns of drug use within
populations; educate clinicians and patients on proper drug usage; and evaluate health-outcome, quality-of-care, cost, and utilization data.

26 Critical Decisions in Emergency Medicine


commonly implicated drugs in interactions. The involvement of a that come with treating the ever-
emergency department visits. Warfarin, pharmacist in the prescribing process growing geriatric population, including
insulin, oral antiplatelets, and oral has been shown to decrease ADEs comorbidities and polypharmacy.
hypoglycemic agents are responsible for and medication errors.19 Moreover, Emergency care providers can take
most ADEs requiring hospitalization. pharmacists can be an extra resource small and reasoned steps to minimize
The combined hospitalization rate for for patient education by providing the risk of potentially dangerous
this group of drugs is 17 times higher information about medications, their ADEs by being aware of common
than the rate for all drugs on the Beers side effects, and possible interactions.19 drug interactions, educating patients
List.15 These medications account for Before prescribing any new drug about the importance of medication
50% of all ADE-related emergency to a geriatric patient, several steps compliance, and maintaining a high
department visits, despite being should be taken to prevent potential index of suspicion when evaluating any
prescribed to only 9% of outpatients.15 ADEs. Consider nondrug therapies geriatric patient.
Eighty-eight percent of such as physical therapy or massage
hospitalizations for ADEs are associated therapy for pain issues. Assume that all REFERENCES
with hematological, endocrine, medications have potential side effects. 1. US Census Bureau, 2004. US interim projections by
age, sex, race, and Hispanic origin. www.census.gov/
cardiovascular, central nervous system, Consider the patient’s physiological population/projections/data/national/usinterimproj.
and anti-infective agents.15 ADEs age and how it affects the ability to html. Accessed June 6, 2015.
2. Blanda M. Pharmacology in the elderly. In: Kahn
stemming from the use of digoxin are absorb, metabolize, and eliminate the JH, Magauran BG Jr, Olshaker JS, eds. Geriatric
the third most common reason for drug. Take a few minutes to assess Emergency Medicine: Principles and Practice.
Cambridge, UK: Cambridge University Press; 2014.
hospitalization; in fact, the exclusion the patient’s current medication list
3. Alexander-Magalee MA. Addressing pharmacology
of digoxin cuts the hospitalization rate to look for common high-risk drugs challenges in older adults. Nursing. 2013
Oct;43(10):58-60.
associated with the Beers List in three.15 with potential interactions. Explain
4. Vestal RE. Aging and pharmacology. Cancer. 1997
At this time, limited data exist about to the patient the indication and most Oct 1;80(7):1302-1310.
ADEs relating to newer anticoagulants, common side effects associated with 5. Fialová D, Onder G. Medication errors in elderly
people: contributing factors and future perspectives.
as compared to warfarin. each medication, as well as the risks of Br J Clin Pharmacol. 2009 Jun;67(6):641-645.
noncompliance. 6. Gallagher P, Barry P, O’Mahony D. Inappropriate
CRITICAL DECISION Start at the lowest possible dosage, prescribing in the elderly. J Clin Pharm Ther. 2007
Apr;32(2):113-121.
What steps can be taken to increasing it as necessary and as 7. McLean AJ, Le Couteur DG. Aging biology and

prevent adverse drug events? tolerated. Create a clear plan for geriatric clinical pharmacology. Pharmacol Rev. 2004
Jun;56(2):163-184.
primary care follow-up, and instruct 8. Hughes SG. Prescribing for the elderly patient: why
Many things can be done both
the patient to have a low threshold for do we need to exercise caution? Br J Clin Pharmacol.
institutionally and in the emergency 1998 Dec;46(6):531-533.
adverse signs and symptoms and to
department to prevent ADEs in the 9. Parkin DM, Henney CR, Quirk J, Crooks J. Deviation
return to the emergency department for from prescribed drug treatment after discharge from
geriatric patient population. Hospitals hospital. Br Med J. 1976 Sep 18;2(6037):686-688.
any possible interactions.
can can help identify potential risks by 10. Vogt-Ferrier N. Older patients, multiple comorbidities,

establishing a computer system to flag Summary polymedication… should we treat everything? Eur
Geriatr Med. 2011 Feb;2(1):48-51.
duplicate prescriptions and potential The elderly constitute one of the 11. Lazarou J, Pomeranz BH, Corey PN. Incidence of
adverse drug reactions in hospitalized patients: a
drug interactions/reactions based on fastest growing segments of our society. meta-analysis of prospective studies. JAMA. 1998
physiological factors such as kidney It is important to understand the body’s Apr 15;279(15):1200-1205.

disease.19,20 changing physiology due to the aging 12. Walker J, Wynne H. Review: the frequency and severity
of adverse drug reactions in elderly people. Age
For example, a hospital in Salt Lake process, and to recognize the challenges Ageing. 1994 May;23(3):255-259.

City established a computer program


called the HELP system (Health
Evaluation through Logical Processing),
which monitors for clinical signs and
symptoms indicative of possible ADEs.18
These red flags, as determined by the
program, include rash, changes to vital n Failing to consider OTC drugs and herbal medications as possible culprits for
signs, seizures, anaphylaxis, diarrhea, ADEs.
fever, and others.18 n Overlooking a patient’s current prescription list and recent changes in
Moreover, medication reconciliation medications.
with the patient is an important process n Discounting a geriatric patient’s changing physiology, which may affect the way
that can be initiated during every visit. drugs are metabolized, leading to side effects and adverse reactions.
A safety mechanism can be established n Administering a high drug dosage to a geriatric patient, rather than starting with
in which the pharmacist must approve a low dosage and slowly increasing it.
certain drugs that are prone to adverse

May 2018 n Volume 32 Number 5 27


CASE RESOLUTIONS
■ CASE ONE intervention. Her INR was reversed, and ■ CASE THREE
the hematoma stabilized without further The 85-year-old woman with
The elderly woman with
treatment.
abdominal and back pain was abdominal pain and black, tarry
tachycardic on presentation. After ■ CASE TWO stool was noted to have a history
reviewing her medication list, the The elderly diabetic man with slurred of osteoarthritis. Her medication
emergency clinician believed the speech and hypoglycemia improved list included meloxicam, and
most likely explanation for her transiently with the administration of she reported that she had also
symptoms was an adverse reaction dextrose 50%. He had a prescription been taking OTC ibuprofen for
caused by ciprofloxacin and for glipizide, a sulfonylurea, which is the last few weeks. The clinician
warfarin. (Ciprofloxacin affects the known to cause hypoglycemia. A review suspected that the drugs (both
hepatic metabolism of warfarin and of his current medication list, indicated NSAIDs) had led to a worsening
increases the circulating level.) that the patient had recently started upper-GI bleed.
A CT scan of her abdomen and taking SMZ/TMP for lower-extremity An upright chest radiograph
pelvis with IV contrast revealed a cellulitis. The emergency clinician was negative for free air. A
large retroperitoneal bleed, with recognized that the concurrent use gastroenterologist was called
no obvious arterial extravasation. of a sulfonylurea and co-trimaxazole
to the emergency department
Vascular surgery and interventional increases the risk of hypoglycemia
to perform an endoscopic
radiology agreed that the patient fourfold. (Sulfamethoxazole inhibits the
examination, which revealed a
should be transferred to a tertiary metabolism of many drugs, including
hospital for further intervention. large duodenal ulcer (3 % 4 cm)
sulfonylurea.)
While waiting for transport, she was with raw borders that were
Due to a rapid drop in his glucose
given IV vitamin K and 2 units of level despite the administration of actively bleeding. There were no
FFP, and continuously monitored. dextrose 50%, the patient required obvious signs of perforation, so
She remained stable and was an IV dextrose 10% drip and was the ulcer was cauterized and the
transferred via advanced life support admitted to the Progressive Care Unit for patient was admitted to the ICU
ambulance. observation and serial fingerstick glucose for further workup. One unit of
The patient was monitored in the tests. He did well and was discharged blood was administered in the
ICU and did not require any surgical the next day. ICU.

13. Klein LE, German PS, Levine DM, Feroli ER Jr, Ardery J.


Medication problems among outpatients. A study
with emphasis on the elderly. Arch Intern Med. 1984
Jun;144(6):1185-1188.
14. Budnitz DS, Pollock DA, Weidenbach KN, Mendelsohn
AB, Schroeder TJ, Annest JL. National surveillance of
emergency department visits for outpatient adverse
drug events. JAMA. 2006 Oct 18;296(15):1858-1866.
15. Budnitz DS, Shehab N, Kegler SR, Richards CL.
Medication use leading to emergency department visits
for adverse drug events in older adults. Ann Intern Med.
2007 Dec 4;147(11):755-765.
16. American Geriatrics Society 2015 Beers Criteria Update
Expert Panel. American Geriatrics Society 2015 Updated
Beers Criteria for Potentially Inappropriate Medication
Use in Older Adults. J Am Geriatr Soc. 2015 Nov ;63(11):
2227-2246.
17. Caterino JM, Emond JA, Camargo CA Jr. Inappropriate
medication administration to the acutely ill elderly: a
nationwide emergency department study, 1992-2000.
J Am Geriatr Soc. 2004 Nov;52(11):1847-1855.
18. Evans RS, Pestotnik SL, Classen DC, Horn SD, Bass
SB, Burke JP. Preventing adverse drug events in
hospitalized patients. Ann Pharmacother. 1994
Apr;28(4):523-527.
19. Fairbanks RJ, Rueckmann EA, Kolstee KE, et al. Clinical
Pharmacists in Emergency Medicine. Agency for
Healthcare Research and Quality. http://www.ahrq.gov/
downloads/pub/advances2/vol4/advances-fairbanks_20.
pdf. Accessed March 2016.
20. Geriatric Emergency Department Guidelines. American
College of Emergency Physicians, The American
Geriatrics Society, Emergency Nurses Association, and
the Society for Academic Emergency Medicine. 2013.
21. Paauw D. Drug Interaction in the Elderly: Beware the
Antibiotics. Medscape. http://www.medscape.com/
viewarticle/848870_2. Accessed March 2016.

28 Critical Decisions in Emergency Medicine


A 55-year-old man with palpitations and lightheadedness.

The Critical ECG


Ventricular tachycardia (VT), rate 190. The differential diagnosis of a wide QRS By Amal Mattu, MD, FACEP
complex tachycardia includes VT, supraventricular tachycardia (SVT) with aberrant Dr. Mattu is a professor, vice chair, and
director of the Emergency Cardiology
conduction (eg, bundle branch block), and sinus tachycardia (ST) with aberrant
Fellowship in the Department of
conduction. In the absence of an obvious and repeating P-QRS pattern, ST can Emergency Medicine at the University
be excluded. This patient’s ECG demonstrates several features that exclude the of Maryland School of Medicine in
Baltimore.
diagnosis of SVT and confirm VT: a taller left “rabbit ear” morphology of the QRS
complex in lead V1, S > R in lead V6, atrioventricular dissociation, and the presence of fusion complexes. Even in the absence
of these diagnostic features, however, VT should always be preferentially chosen and treated. Treating an SVT as if it were VT is
generally safe; however, if VT is mistakenly diagnosed and treated as SVT, the results can be deadly.

a) Right bundle branch block–type of à Features suggestive of VT in this b)


morphology with RsR’ configuration. wide QRS complex tachycardia.
Note that the R wave is larger than R’ Wave
a)
the R’ wave (taller left “rabbit ear”),
R Wave
consistent with VT.
b) The R wave is larger than the R’
wave in the QRS complex, a finding
consistent with VT.
c) Fusion beats (a combination of
supraventricular and ventricular
beats, indicated by arrows) strongly c) R > R’ Wave
suggestive of VT.

From Mattu A, Brady W, ECGs for the Emergency Physician


2. London: BMJ Publishing; 2008:11,23. Reprinted with
permission.

May 2018 n Volume 32 Number 5 29


CME
Reviewed by Lynn Roppolo, MD, FACEP

Qualified, paid subscribers to Critical Decisions in Emergency Medicine may receive


CME certificates for up to 5 ACEP Category I credits, 5 AMA PRA Category 1
Credits™, and 5 AOA Category 2-B credits for completing this activity in its entirety.

QUESTIONS Submit your answers online at acep.org/newcriticaldecisionstesting; a score of 75%


or better is required. You may receive credit for completing the CME activity any
time within 3 years of its publication date. Answers to this month’s questions will be
published in next month’s issue.

1 Which of the following is a feature of the pediatric


bony skeleton when compared to adult bones? 5 A 2-month-old boy is brought in for the evaluation of
right-leg pain. He parents explain that he rolled off a
table at home. He is fussy, and swelling and bruising are
A. Pediatric bones are less elastic
B. Pediatric bones are more porous noted on the right thigh. An x-ray reveals a midshaft
C. Pediatric bones are strongest at the physis femur fracture. He has no other signs of injury. What is
D. Pediatric patients are less likely to sustain bowing the most appropriate next step in his management?
fractures A. Closed reduction in the emergency department
followed by casting

2
An 8-year-old boy presents with left-knee pain and
a limp. He has had subjective fevers at home, but
denies trauma. His temperature is 38.8°C (101.8°F);
B. Skeletal survey, blood work, and orthopedic
consultation
C. Orthopedic consultation for spica casting
his left leg has significant warmth, swelling, and D. Posterior long-leg splint and orthopedic follow-up
tenderness with restricted active and passive
range of motion. He is unable to bear weight. His
laboratory studies show a WBC count of 25,000 6 Where is the location of pain in a patient with Osgood-
Schlatter disease?
cells/hpf with 90% neutrophils, and a CRP of A. Calcaneal apophysis
12.0 mg/dL. Orthopedic surgery is consulted, and B. Inferior pole of the patella
a joint aspirate is obtained. What organism is most C. Posterior aspect of the medial tibial epicondyle
likely to grow on culture? D. Tibial tuberosity

7
A. Neisseria gonorrhoeae What is the most sensitive and specific imaging modality
B. Pseudomonas aeruginosa for the evaluation of bone and joint infections?
C. Salmonella enterica A. CT
D. Staphylococcus aureus B. MRI
C. Plain films
3
A 9-year-old boy presents with bilateral heel pain
and limping, which has worsened since basketball
D. Ultrasound

8
season started 1 month ago. He is in no acute A 16-year-old boy presents with a right tibial shaft
distress, but has moderate tenderness over fracture caused by a direct blow sustained during a
the Achilles’ tendon insertion to the calcaneus basketball game. In the emergency department, he
bilaterally. What is the most appropriate next step becomes increasingly agitated, asks to go home, and
in his management? has worsening pain, requiring IV morphine. He has
A. Discharge home with rest, scheduled NSAIDs, and right lower-extremity swelling, point tenderness, and
primary care physician follow-up worsened pain with passive movement of his foot. His
B. Obtain blood work, including a CBC, a CRP, an distal pulses are intact, and he has normal sensation to
ESR, and a blood culture the foot. However, he is unable to ambulate secondary
C. Obtain an MRI of the bilateral feet to worsening pain. What next steps should be taken?
D. Obtain an urgent orthopedic consultation A. Admit to the hospital for serial examinations
B. Initiate an emergent orthopedic consultation

4 Which of the following is characteristic of Legg-


Calvé-Perthes (LCP) disease?
C. Initiate posterior long-leg splinting and routine
orthopedic surgery referral
A. It carries a low risk for subsequent fracture D. Provide a repeat dose of morphine and continue to
B. It commonly occurs in adolescents observe
C. It often responds to conservative therapy
D. Most cases are bilateral on presentation

30 Critical Decisions in Emergency Medicine


9 Which of the following is an advantage of CT
when compared to MRI?
A. CT generally does not require sedation
15 Which drugs are responsible for the largest
number of adverse drug reactions in geriatric
patients?
B CT has improved sensitivity for detecting A. Glipizide, insulin, and aspirin
inflammatory conditions B. Insulin, hydralazine, and clonidine
C. CT is more easily interpreted by radiologists C. Warfarin, amiodarone, and insulin
D. CT provides lower levels of ionizing radiation D. Warfarin, insulin, and digoxin

10 Which of the following is characteristic of a


toddler’s fracture?
A. Immobilization is not necessary in most cases
16 Which of the following strategies is unlikely to
decrease adverse drug reactions in the acute
setting?
B. Plain films are often normal on initial A. Computer system safety triggers
evaluation B. Consultation with an emergency department
C. Significant force is required to cause this type pharmacist
of injury C. Implementing protocols that allow ancillary staff
D. The most common mechanism is a direct to place medication orders on your behalf
blow to the tibia D. Medication reconciliation

11 Which of the following is an expected


physiological change in the geriatric
population? 17 How can ciprofloxacin affect warfarin?
A. Decreases its clearance
A. Increased GFR and creatinine B. Decreases its effectiveness
B. Increased adipose tissue C. Increases its clearance
C. Increased gastric motility D. Increases pharmacodynamics
D. Increased lean body weight

12 Which of the following describes


pharmacokinetics?
18 Which of the following factors affects
pharmacokinetics?
A. Cardiac function
A. The means by which drugs are eliminated B. GI function
B. The means by which drugs are metabolized C. Liver function
C. The means by which the body interacts with D. Pulmonary function
medications

19
D. The way medications affect one another What can pharmacists do to help prevent ADEs?
A. Arrange follow-up with a primary care physician

13
How are adverse drug events (ADEs) best
B. Arrange home health care
defined?
C. Double check both inpatient and outpatient
A. Any drug-related side effects or interactions medication orders
B. Events that require drug intervention D. Prescribe medications on the treating
C. Mortality from drug interactions physician’s behalf
D. Taking too many medications at once

14 Which of the following drug classes contribute


to the majority of ADEs in geriatric patients? 20
What is the risk of ADEs in a patient taking seven
or more prescribed medications?
A. 27%
A. Antiarrythmic medications and
anticoagulants B. 54%
B. Anticoagulants and diabetic medications C. 82%
C. Antihypertensive medications and D. 98%
anticoagulants
D. Diabetic medications and antihypertensive
medications

ANSWER KEY FOR APRIL 2018, VOLUME 32, NUMBER 4


1 2 3 4 5 6 7 8 9 10 11 12 13 14 15 16 17 18 19 20
D A D D C C A A A D C C A D B B A A D D

May 2018 n Volume 32 Number 5 31


Drug Box Tox Box
BIOTIN (VITAMIN B7) AMNESTIC SHELLFISH POISONING
By Frank LoVecchio, DO, MPH, FACEP, By Richard A. Koch, MD, FACEP; and Christian A. Tomaszewski, MD, MS, MBA,
Maricopa Medical Center/Banner Poison and FACEP, University of California, San Diego
Drug Center, Phoenix, AZ Amnestic shellfish poisoning results from consuming mussels and clams containing
Biotin is a water-soluble vitamin found in domoic acid. Even marine animals can succumb to the neurotoxin’s effects. Alfred
common foods such as nuts, whole grains, Hitchcock’s classic thriller, The Birds, is said to have been inspired by a 1961
fruits, and vegetables. It also is found in multi- outbreak.
and prenatal vitamins and dietary supplements
marketed for hair, skin, and nail growth. Mechanism
Domoic acid is a tasteless, odorless, heat-stable, water-soluble, polar, nonprotein
Laboratory Abnormalities amino acid mainly produced by Pseudo-nitzschia diatoms. Filter-feeding shellfish
High doses can interfere with hundreds of (eg, mussels, clams) accumulate the toxin. It is excitatory at mammalian glutamate
common blood tests. Of particular note, receptors, leading to excessive influx of Ca2+ and Na2+ into neurons and apoptosis.
troponin levels can be either falsely low or Presentation
high. The US Food and Drug Administration Usually manifests <5 hours (up to 38 hours) after shellfish ingestion. Most patients
has reported an increase in the number of report initial gastrointestinal symptoms followed by neurological symptoms:
adverse events related to biotin interference • Abdominal cramps, nausea, vomiting, diarrhea
in laboratory tests, including one death. • Confusion, memory loss, disorientation
• Hemiparesis, ophthalmoplegia, purposeless chewing, grimacing
Vitamin B7 Deficiency
• Seizures, status epilepticus, cranial nerve palsies, coma
Although biotin deficiency is fairly rare,
• Cardiac dysrhythmias and/or hemodynamic instability
some patients (eg, pregnant women) may
Mortality ≤2%, usually in older patients. Seizures resolve over weeks, but <10%
experience mild forms. The excessive
have permanent neurological problems.
consumption of raw eggs can also lead to
a deficiency. (Raw egg whites contain a Evaluation
protein called avidin, which binds to biotin • Electrolyte levels to assess for losses due to vomiting and diarrhea
and prevents its absorption. The protein is • Orthostatic vital signs to assess for fluid loss or direct toxic effect
deactivated during the cooking process.) • Electrocardiogram to rule out other causes of cardiac dysrhythmias
• Computed tomography to rule out other causes of neurological injury
Mechanism • Enzyme-linked immunosorbent assay or high-performance liquid
Biotin-containing enzymes participate in chromatography testing to test shellfish
important metabolic pathways such as the Treatment
production of glucose and fatty acids. Supportive care includes IV fluids, vasopressors for hypotension, antiemetics,
and the correction of electrolyte abnormalities. Benzodiazepines, propofol,
Dosing
and barbiturates for seizures. The benefit of activated charcoal is unclear
The recommended daily intake of biotin
(contraindicated if altered consciousness).
for men, women, and pregnant females is
30 mcg. Disposition
Abnormal blood tests have been reported • Patients with an unclear diagnosis, significant neurological symptoms, or
in patients taking high levels (eg, ≥300 mg electrolyte/fluid loss require further treatment or evaluation.
per day). • Patients with mild symptoms can be discharged with follow-up care.

You might also like